Old Q&A – Infectious Disease

  A 21-year-old woman comes to the university health clinic complaining of a 2-week history of fatigue, lethargy, and fever. She has also noticed a mild sore throat. Her past medical history is otherwise unremarkable and she takes only oral contraceptive pills for birth control and acne. Her temperature is 39.0 C (100.4 F), blood pressure is 120/75 mm Hg, pulse is 82/min, and respirations are 18/min. She appears somewhat ill, but in no clear distress. Her pharynx appears erythematous and she has mild splenomegaly. Supportive therapy and avoidance of contact sports is the appropriate treatment if laboratory studies show

Top of Form

  A. leukopenia with atypical leukocytosis
  B. positive culture for group A beta-hemolytic Streptococcus
  C. positive Mycoplasma PCR
  D. positive RNA p24 antigen PCR
  E. positive serum HSV PCR

Bottom of Form

Explanation:

The correct answer is A. This patient has infectious mononucleosis caused by the Epstein-Barr virus. This disease can present in a very similar manner to infectious pharyngitis caused by the group A beta hemolytic streptococcus. These patients typically present with more mild symptoms of fever and sore throat and in many cases have splenomegaly. The classic laboratory finding is leukopenia with atypical leukocytosis. The treatment of mono typically includes supportive therapy with avoidance of contact sports (to avoid splenic rupture).

A positive culture for group A beta-hemolytic Streptococcus (choice B) would represent infection with a bacteria that causes bacterial pharyngitis, not mononucleosis. Although the presentations of the 2 diseases may be similar, their causative agents are quite dissimilar. Antibiotics are used to group A beta-hemolytic Streptococcus.

A positive mycoplasma PCR (choice C) represents infection with Mycoplasma pneumonia, a common cause of atypical community acquired pneumonia in this age group. Antibiotics are used to treat this infection.

A positive RNA p24 antigen PCR (choice D) would represent evidence of infection with the HIV virus. Antiviral therapy is used to treat HIV.

A positive serum HSV PCR (choice E) indicates active herpes virus infection. These agents are responsible for a number of human diseases, including pharyngitis, but the symptoms are much milder. Herpes virus infection does not cause infectious mono. Antiviral therapy is used to treat HSV.

 

A 3-year-old girl is brought to the office by her father because a boy in her daycare center was diagnosed with group A meningococcal meningitis. You care for the boy that they are referring to so you know that this information is accurate. Even though this girl is asymptomatic, the father is very concerned about her health. Her physical examination is completely unremarkable. The most appropriate next step in management is to

Top of Form

  A. administer a serogroup-specific quadrivalent meningococcal vaccine
  B. administer a single dose of ceftriaxone, intramuscularly
  C. admit her to the hospital for careful observation
  D. obtain throat and nasopharyngeal culture to decide if treatment is indicated
  E. recommend careful observation and schedule a follow-up visit in 1 week

Bottom of Form

Explanation:

The correct answer is B. The risk of contracting meningococcal disease among household members and childcare and nursery school contacts is considered high enough to warrant chemoprophylaxis. Rifampin, ceftriaxone, and ciprofloxacin are the 3 recommended agents used as chemoprophylaxis for invasive meningococcal disease.

A serogroup-specific quadrivalent meningococcal vaccine (choice A) is used to prevent cases of invasive meningococcal disease. It is given routinely to all military recruits and recommended for children with asplenia and other immunodeficiencies. Some groups also advise college students to receive this vaccine. The vaccine may be useful as an adjunct to chemoprophylaxis during an outbreak, but it is not routinely recommended, as chemoprophylaxis is.

It is unnecessary to admit her to the hospital for careful observation (choice C). This patient should be given chemoprophylaxis and her father should be advised to observe her carefully for the development of a febrile illness. Medical evaluation should be sought immediately if an illness occurs, but hospitalization at this time in this asymptomatic patient is not indicated.

Throat and nasopharyngeal culture to decide if treatment is indicated (choice D) is incorrect. Throat and nasopharyngeal cultures are not useful in determining the risk of contracting this disease. Chemoprophylaxis for invasive meningococcal disease is indicated for this high-risk contact.

It is inappropriate to recommend careful observation and schedule a follow-up visit in 1 week (choice E). This patient is a high-risk contact and therefore requires chemoprophylaxis. Careful observation is important too.

 

A 72-year-old man with non-Hodgkin’s lymphoma, who is 10 days post chemotherapy, has persistent fevers. Of note, 3 days after his chemotherapy finished, he had a temperature of 38.5 C (101.3 F). He was started on ceftazadime and tobramycin. His fever resolved initially. However, now it is 7 days later and he again has similar temperature elevations. He has also developed some minimal hemoptysis. His blood pressure is 115/85 mm Hg, pulse 82/min, and respirations 20/min. Heart has a regular rhythm with no murmurs, lungs have some dry basilar crackles, abdomen is benign, and extremities have 1+ edema, but no erythema. Laboratory studies show a leukocyte count of 3,200mm3, hematocrit 28%, and platelets 18,000mm3. A chest x-ray shows development of some bilateral nodular densities. A CT scan of the lungs confirms multiple lung nodules, many of which have small hazy borders consistent with minimal perinodular hemorrhage. The most appropriate next step in management is to

Top of Form

  A. add amphotericin B intravenous therapy to his current therapy
  B. change the chemotherapy regimen due to treatment failure
  C. continue the ceftazidime and tobramycin and give it time to work
  D. refer the patient to radiation oncology for emergent bilateral lung radiation
  E. send him for a transesophageal echocardiogram

Bottom of Form

Explanation:

The correct answer is A. In any post chemotherapy patient who has persistent fever despite appropriate broad-spectrum antibiotic coverage, fungal and resistant Gram-positive organisms need to be considered. This patient also has a chest CT scan showing multiple bilateral lung nodules with surrounding hemorrhage which is classic for invasive aspergillosis. The most effective current therapy is with amphotericin B. Continuing the ceftazadime and tobramycin (choice C) alone would not address the current problem.

The patient is presenting with an acute infectious process related to chemotherapy induced immune suppression. Although a lymphoma can involve the lung directly, it is not particularly common and does not fit this clinical scenario. This makes a treatment failure (choice B) incorrect. Bilateral lung irradiation is not something that is typically done in non-Hodgkin’s lymphoma and certainly would not be an emergent procedure (choice D).

A transesophageal echocardiogram (choice E) can be used to look at vegetations on heart valves seen in endocarditis. If the vegetations are on the tricuspid or pulmonary valves, they can give off septic emboli to the lungs. This too can cause multiple lung nodules. However, they are often cavitary and do not show the perinodular hemorrhage that is classic for invasive aspergillosis.

 

A 72-year-old man with non-Hodgkin’s lymphoma, who is 10 days post chemotherapy, has persistent fevers. Of note, 3 days after his chemotherapy finished, he had a temperature of 38.5 C (101.3 F). He was started on ceftazadime and tobramycin. His fever resolved initially. However, now it is 7 days later and he again has similar temperature elevations. He has also developed some minimal hemoptysis. His blood pressure is 115/85 mm Hg, pulse 82/min, and respirations 20/min. Heart has a regular rhythm with no murmurs, lungs have some dry basilar crackles, abdomen is benign, and extremities have 1+ edema, but no erythema. Laboratory studies show a leukocyte count of 3,200mm3, hematocrit 28%, and platelets 18,000mm3. A chest x-ray shows development of some bilateral nodular densities. A CT scan of the lungs confirms multiple lung nodules, many of which have small hazy borders consistent with minimal perinodular hemorrhage. The most appropriate next step in management is to

Top of Form

  A. add amphotericin B intravenous therapy to his current therapy
  B. change the chemotherapy regimen due to treatment failure
  C. continue the ceftazidime and tobramycin and give it time to work
  D. refer the patient to radiation oncology for emergent bilateral lung radiation
  E. send him for a transesophageal echocardiogram

Bottom of Form

Explanation:

The correct answer is A. In any post chemotherapy patient who has persistent fever despite appropriate broad-spectrum antibiotic coverage, fungal and resistant Gram-positive organisms need to be considered. This patient also has a chest CT scan showing multiple bilateral lung nodules with surrounding hemorrhage which is classic for invasive aspergillosis. The most effective current therapy is with amphotericin B. Continuing the ceftazadime and tobramycin (choice C) alone would not address the current problem.

The patient is presenting with an acute infectious process related to chemotherapy induced immune suppression. Although a lymphoma can involve the lung directly, it is not particularly common and does not fit this clinical scenario. This makes a treatment failure (choice B) incorrect. Bilateral lung irradiation is not something that is typically done in non-Hodgkin’s lymphoma and certainly would not be an emergent procedure (choice D).

A transesophageal echocardiogram (choice E) can be used to look at vegetations on heart valves seen in endocarditis. If the vegetations are on the tricuspid or pulmonary valves, they can give off septic emboli to the lungs. This too can cause multiple lung nodules. However, they are often cavitary and do not show the perinodular hemorrhage that is classic for invasive aspergillosis.

 

  Two separate tests for the detection of anthrax exposure have recently been developed, a general screening test and a confirmatory diagnostic test. The screening test is used in the general population, while the diagnostic test is used to confirm suspected cases already identified by other means. Non-physician field personnel administer the screening test, while physicians or technicians under physician supervision administer the diagnostic test. The screening test is much cheaper than the diagnostic test, costing only 1/10 as much. Although both have the same level of sensitivity, over the course of their use, the diagnostic test is discovered to have a substantially higher positive predictive value. This difference between the two tests is most likely accounted for by the

Top of Form

  A. care of the laboratory technicians
  B. differences in specificity between the tests
  C. expense of the test
  D. length of time the tests have been used
  E. prevalence within the sample tested
  F. training of the personnel administering each test.
  G. uncategorized error variance

Bottom of Form

Explanation:

The correct answer is E. The prevalence of exposure in the general population is almost certainly much less than the prevalence in the population to which the diagnostic test was applied. Higher prevalence enhances positive predictive value. Simply put, if more people actually have the disease, then a positive test result has a higher probability of being true. Note that for the reasons described here, diagnostic tests always have a higher positive predictive value than screening tests.

Although the care and skill of laboratory technicians (choice A) varies, there is no evidence that this is a key issue here.

If the diagnostic test has better specificity (choice B) than the screening test, we would expect a higher positive predictive value. However, while this is possible, the different sample prevalence is almost a certainty making that the better answer.

More expensive (choice C) does not, by itself, mean better.

The question gives no information about the length of time (choice D) that either test has been used.

The assumption that non-physicians (choice F) provide less accurate test results is supported by neither the question content nor real life experience.

Every test result has error as a part of the measurement it provides, but that is not what accounts for the differences reported here. The difference in results here are the result, not of uncategorized error (choice G), but error rates linked directly to the different prevalences on which the tests were conducted.

 

  A 5-year-old boy is brought to the clinic by his mother because of the new onset of a flaky scalp and patches of hair loss. He just started preschool 2 months ago and his teacher noted the alopecia during a nap break. His past medical and birth history are insignificant, and he is not on any medications at this time. He has 1 cat and 1 dog at home. On examination, there are multiple circular patches of alopecia studded with black dots on the surface of the scalp. After examining the boy, the mother shows you lesions on her right shoulder. There is an annular erythematous plague with central clearing. The edge is slightly raised and there are tiny vesicles and a fine scale. There is mild lymphadenopathy appreciated. The best next diagnostic step is to

Top of Form

  A. exam the boy’s hair microscopically with potassium hydroxide
  B. inquire about autoimmune diseases in the family and obtain a thyroid function test
  C. perform a punch biopsy of the boy’s scalp
  D. perform a Wood’s light exam of the boy’s scalp
  E. refer him to psychiatry for trichotillomania

Bottom of Form

Explanation:

The correct answer is A. Examining the hair microscopically with potassium hydroxide (KOH) is correct because this child has tinea capitis secondary to his mother’s tinea corporis. Tinea capitis commonly presents as areas of alopecia with studded black dots representing infected hairs broken off at or below the surface of the scalp. Different species of fungus can cause either inflammatory or non-inflammatory lesions. Trichophyton mentagrophytes, the species responsible for both inflammatory tinea capitis and tinea corporis, can result in pain and regional lymphadenopathy. Most commonly, the mode of transmission is child to child in school age children. The fact that this child’s mom has classic “ringworm” on her left shoulder, which is usually described as concentric erythematous plaque with central clearing and superficial scales points the differential diagnosis toward a fungal infection. Diagnosis of tinea capitis is usually confirmed with microscopic exam of the hair and looking for ectothrix or endothrix spores.

Inquiring about autoimmune disease (choice B) is incorrect because autoimmune processes are usually associated with alopecia areata, not tinea capitis. Alopecia areata is not usually associated with lymphadenopathy or “black dots” of broken hair.

Punch biopsy of the scalp (choice C) is incorrect because if tinea capitis is suspected, the first step toward diagnosis includes KOH of hair and a fungal culture of the scalp.

Wood’s light exam of the scalp (choice D) is incorrect because only ectothrix species fluoresce with Wood’s light but not endothrix types. Therefore, this is not the best diagnostic method to confirm tinea capitis

Referral to psychiatry for trichotillomania (choice E) is incorrect, because this type of alopecia should not have associated lymphadenopathy.

 

A 62-year-old man underwent coronary artery bypass surgery 12 weeks ago. His postoperative course is complicated by bleeding and cardiac tamponade, for which he had an emergency sternotomy. One week later he developed a sternal wound infection for which debridement and a pectoralis muscle flap procedure is performed. During these surgeries he receives multiple blood transfusions. At his 3-month followup he reports dark urine, fatigue, and anorexia. On examination, he is not jaundiced. There is mild, tender hepatomegaly on palpation of the abdomen. Laboratory studies show:

The most likely explanation for the patient’s clinical condition is

Top of Form

  A. acute viral hepatitis A
  B. acute viral hepatitis B
  C. acute viral hepatitis C
  D. acute viral hepatitis D
  E. acute viral hepatitis E

Bottom of Form

Explanation:

The correct answer is C. Post transfusion non-A, non-B hepatitis is mostly the result of hepatitis C infection. The incubation period is usually 5-10 weeks and the mean peak aminotransferase levels are 500-1000 IU/L. Anti-HCV antibody is commonly not detectable until 18 weeks after illness onset. Approximately 70% of patients with acute hepatitis C progress to chronic hepatitis and potentially cirrhosis.

Negative hepatitis A (choice A) antibody rules out hepatitis A infection.

Negative serology for hepatitis B (choice B) rules out hepatitis B infection.

Hepatitis D (delta) virus (choice D) is capable of infecting only patients who also have HBsAg, because HDV is an incomplete RNA virus.

Hepatitis E virus (choice E) is rare, except in association with water-borne epidemics in India, the Middle East, and South America.

 

An 8-year-old boy with asthma is admitted to the hospital with shortness of breath. The mother tells you that he is usually well controlled with bronchodilator inhalers. However, for the past 2 days he has had rhinorrhea, a low-grade fever, and myalgias. She also reports that the child has a non-productive cough. Bronchodilators temporarily improved the child’s breathing at home, but it once again worsened and they became worried. On admission, this child is given droplet precautions. Nebulized bronchodilator treatments are initiated. Oxygen supplementation is given by nasal cannula and he is given aspirin for the relief of fever. For prophylaxis of influenza, amantadine is administered because of a recent influenza outbreak in the community. Of all of the therapies initiated in this patient, the one that is not indicated in this case is

Top of Form

  A. amantadine
  B. aspirin
  C. droplet precautions
  D. nebulized bronchodilators
  E. oxygen

Bottom of Form

Explanation:

The correct answer is B. Although aspirin is appropriate for the relief of fever, this patient is an 8-year-old child. The use of aspirin in a child with influenza is contraindicated, due to the association of aspirin usage in children with influenza and the development of Reye’s syndrome. Reye’s syndrome is more common with influenza B, but outbreaks have been found to be associated with influenza A. Hence, during an outbreak of influenza, aspirin is contraindicated in children with respiratory symptoms or fever.

Amantadine until 48 hours after the symptoms have resolved (choice A) is a therapy for the prophylaxis of influenza.

Placing the child on droplet precautions is essential for the prevention of spread of respiratory infections during an influenza outbreak (choice C).

Nebulized bronchodilators (choice D) are essential for preventing bronchospasm and for better oxygenation in children with respiratory symptoms.

Supplementation of oxygen, if oxygen saturation is low (choice E), is an essential therapeutic maneuver in children with respiratory symptoms.

 

A 71-year-old woman comes to the emergency department because of severe shortness of breath, retrosternal chest pain, a fever, and a dry cough that has worsened over the past three weeks. She says that she is rarely sick and she prides herself on being the “healthiest and most active grandmother in the northeast.” She swims everyday and goes out with friends four nights a week since her husband passed away five years ago. She blushes as she admits that she has many male “suitors”. She does not smoke cigarettes. However, she drinks a “moderate” amount of alcohol each day. She recalls having an episode of fever, headaches, joint pain, a loss of appetite, and a mild sore throat a few months ago that she did not seek medical attention for because she assumed it was a “virus”. Her temperature is 38.8 C (101.8 F) and respirations are 35/min. She has bibasilar rales and significant cervical, axillary, and inguinal lymphadenopathy. A chest x-ray shows bilateral patchy alveolar infiltrates. Histologic evaluation of a sputum sample obtained by bronchoalveolar lavage shows round structures when stained with methenamine silver. An important question to ask at this time is:

Top of Form

  A. “Did you or your late husband ever install insulation or brake lining, do construction work, or work in a shipyard?”
  B. “Do you engage in unprotected sexual intercourse at the present time or at any other time in the past?”
  C. “Have you ever been involved in a homosexual relationship?”
  D. “Have you ever had a morning drink to get started (an “eyeopener”)?”
  E. “Have you ever had a positive PPD or been exposed to anyone with tuberculosis?”

Bottom of Form

Explanation:

The correct answer is B. This patient most likely has Pneumocystis carinii pneumonia (PCP), and she is probably infected with the human immunodeficiency virus (HIV), in her case, most likely due to unprotected sexual intercourse with an infected individual. It is important to recognize that elderly patients may be sexually active, even if their spouse is no longer around. They are at risk for HIV and other sexually transmitted diseases because they may not think to use a condom because they are not worried about getting pregnant. PCP is characterized dyspnea, fever, a nonproductive cough, retrosternal chest pain, tachypnea, tachycardia, few abnormalities on auscultation, bilateral patchy alveolar infiltrates on chest x-ray, and round cysts found under light microscopy when stained with methenamine silver. The treatment is trimethoprim-sulfamethoxazole. The “episode of fever, headaches, joint pain, a loss of appetite, and mild sore throat” that she describes having a few months earlier is consistent with the acute HIV syndrome which affects many patients with HIV, approximately three to six weeks after the primary infection. This syndrome coincides with plasma viremia (wide dissemination of the virus). The symptoms gradually subside over a few weeks.

“Did you or your late husband ever install insulation or brake lining, do construction work, or work in a shipyard?” (choice A) is an important question if an asbestos-related disease is suspected, but this case is more consistent with Pneumocystis carinii pneumonia than asbestosis or malignant mesothelioma. Asbestosis is characterized by dyspnea, a nonproductive cough, basilar crackles or rales, clubbing, linear streaking and pleural thickening seen on chest x-ray, and ferruginous bodies seen on microscopic examination of lung tissue (rod-shaped bodies with clubbed ends). Malignant mesothelioma is a tumor of the pleura that is characterized by chest pain, dyspnea, a cough, a chest x-ray showing pleural fluid, irregular pleural thickening, and a biopsy demonstrating the malignant cells.

“Have you ever been involved in a homosexual relationship?” (choice C) is not a vital question at this time because it is very unlikely that this woman contracted HIV and AIDS from unprotected sexual intercourse with another woman. It is more likely that she contracted the infection from unprotected sexual intercourse with a man. Also, if you wanted to know if a patient has homosexual relationships, it is better to ask in a nonjudgmental way, such as, “Do you have sex with men, women, or both?”

“Have you ever had drink in the morning to get started (an “Eyeopener”)?” (choice D) is a part of a four question screening test to detect problem drinking called the CAGE questionnaire. The other three questions that make up the CAGE questionnaire include: “Have you ever felt the need to Cut down on your drinking?”, “Have you ever felt Annoyed by criticisms of your drinking?”, and “Have you ever had Guilty feelings about drinking?”. Two positive responses indicate that a problem is likely. These questions may be important, but the immediate concern in this case is this patient’s Pneumocystis carinii pneumonia and whether she has been infected with the HIV virus. Questions related to her sexual practices may help to identify the source of infection.

“Have you ever had a positive PPD or been exposed to anyone with tuberculosis?” (choice E) is a question that is important if tuberculosis or an aspergilloma is suspected. However, this case is more consistent with Pneumocystis carinii pneumonia. Primary tuberculosis (TB) is characterized by systemic symptoms, a cough, sputum production, hemoptysis, lower lobe infiltrates, hilar node enlargement, pleural involvement, and the presence of acid-fast bacilli. Reactivation TB is characterized by infiltrates with cavitation in the apices. An aspergilloma is a “fungus ball”‘ that typically forms within a preexisting cavity (from TB or sarcoidosis) in the pulmonary parenchyma. The patients may be asymptomatic or present with hemoptysis. A culture shows fungal mycelia, which appear as branching hyphae. In this case, microscopic evaluation shows round structures, not acid-fast bacilli or hyphae.

 

  A 31-year-old alcoholic homeless man with a history of type 1 diabetes comes to the urgent care clinic with a left foot ulcer. The ulcer has been present for 4 months, but has been sore for the past 2 weeks. There is no other medical history. His blood pressure is 92/54 mm Hg and pulse is 170/min. Physical examination shows an eschar extending to the bone with necrotic sides on the dorsal aspect of the hallux just distal to the interphalangeal joint. Laboratory studies reveal a leukocyte count of 33,000/mm3. Fingerstick glucose is 210 mg/dL. Urinalysis, complete blood count, chest radiograph, and blood cultures are pending. An electrocardiogram shows a sinus tachycardia at a rate of 180/min. The most appropriate first-line therapy for this patient’s tachycardia is

Top of Form

  A. atenolol
  B. intravenous imipenem
  C. intravenous 0.9% normal saline and broad spectrum antibiotics
  D. metoprolol
  E. packed red blood cell transfusion

Bottom of Form

Explanation:

The correct answer is C. This patient is at risk for Gram-negative cellulitis because he is diabetic. Clinically, this patient is most likely having sepsis from a cellulitis. He is exhibiting the signs of septic shock, which include tachycardia and hypotension due to decreased vascular tone secondary to circulating toxins. The treatment is intravenous fluids, preferably a 0.9% normal saline which remains in the vasculature longer than a 0.5% normal saline or dextrose in water intravenous solutions. Intravenous fluid will address the issue of septic shock immediately, and broad-spectrum antibiotics are critical to stopping the proliferating infection.

β-blockers, such as atenolol (choice A) will stop the tachycardia, but this patient is most likely tachycardic as a normal response to hypotension. Clinically, this patient is most likely having sepsis from a cellulitis. He is exhibiting the signs of septic shock which include tachycardia and hypotension due to decreased vascular tone secondary to circulating toxins. The treatment is intravenous fluids, preferably a 0.9% normal saline which remains in the vasculature longer than a 0.5% normal saline or dextrose in water intravenous solutions. Since this patient most likely has septic shock, intravenous fluids and antibiotics are the most appropriate therapy. Intravenous fluids will address the issue of septic shock immediately and broad-spectrum antibiotics are critical to stopping the proliferating infection.

Antibiotics (choice B) are necessary for the treatment of this infection, but they do not work quickly enough to prevent the impending septic shock. Intravenous fluids will address the issue of septic shock immediately and broad-spectrum antibiotics are critical to stopping the proliferating infection.

β-blockers, such as metoprolol (choice D) will stop the tachycardia, but this patient is most likely tachycardic as a normal response to hypotension, so intravenous fluids are the most appropriate first line therapy.

Packed red blood cell transfusion (choice E) may be necessary if there is a severe anemia, but saline is the first choice to rehydrate an unstable intravascular volume in a depleted patient.

 

  A 21-year-old woman comes to the university health clinic complaining of a 2-week history of fatigue, lethargy, and fever. She has also noticed a mild sore throat. Her past medical history is otherwise unremarkable and she takes only oral contraceptive pills for birth control and acne. Her temperature is 39.0 C (100.4 F), blood pressure is 120/75 mm Hg, pulse is 82/min, and respirations are 18/min. She appears somewhat ill, but in no clear distress. Her pharynx appears erythematous and she has mild splenomegaly. Supportive therapy and avoidance of contact sports is the appropriate treatment if laboratory studies show

Top of Form

  A. leukopenia with atypical leukocytosis
  B. positive culture for group A beta-hemolytic Streptococcus
  C. positive Mycoplasma PCR
  D. positive RNA p24 antigen PCR
  E. positive serum HSV PCR

Bottom of Form

Explanation:

The correct answer is A. This patient has infectious mononucleosis caused by the Epstein-Barr virus. This disease can present in a very similar manner to infectious pharyngitis caused by the group A beta hemolytic streptococcus. These patients typically present with more mild symptoms of fever and sore throat and in many cases have splenomegaly. The classic laboratory finding is leukopenia with atypical leukocytosis. The treatment of mono typically includes supportive therapy with avoidance of contact sports (to avoid splenic rupture).

A positive culture for group A beta-hemolytic Streptococcus (choice B) would represent infection with a bacteria that causes bacterial pharyngitis, not mononucleosis. Although the presentations of the 2 diseases may be similar, their causative agents are quite dissimilar. Antibiotics are used to group A beta-hemolytic Streptococcus.

A positive mycoplasma PCR (choice C) represents infection with Mycoplasma pneumonia, a common cause of atypical community acquired pneumonia in this age group. Antibiotics are used to treat this infection.

A positive RNA p24 antigen PCR (choice D) would represent evidence of infection with the HIV virus. Antiviral therapy is used to treat HIV.

A positive serum HSV PCR (choice E) indicates active herpes virus infection. These agents are responsible for a number of human diseases, including pharyngitis, but the symptoms are much milder. Herpes virus infection does not cause infectious mono. Antiviral therapy is used to treat HSV.

 

A 43-year-old woman comes to the emergency department because of fever and abdominal pain. She has a history of cirrhosis and long standing alcohol abuse. She takes no medications except for the occasional acetaminophen for a headache. She reports that 5 days ago, she had fever of 38.6 C (101.5 F) and the gradual onset of diffuse abdominal pain. Her blood pressure is 95/40 mm Hg and pulse is 104/min and regular. Physical examination shows clear lungs, numerous spider angiomata on her thorax and back, and a massively distended abdomen with shifting dullness by percussion. An abdominal paracentesis is performed and the results are as follows:

Laboratory studies show:

The most appropriate pharmacotherapy is

Top of Form

  A. ampicillin, intravenously
  B. azithromycin, intravenously
  C. cefotaxime, intravenously
  D. penicillin G, intravenously
  E. vancomycin, intravenously

Bottom of Form

Explanation:

The correct answer is C. The prevalence of infection of ascites fluid, so called spontaneous bacterial peritonitis (SBP), has an estimated prevalence of 50% in hospitalized cirrhotics. ALL febrile patients admitted with ascites must have an abdominal paracentesis performed to both determine the cause for the ascites and rule out infection of the ascites. The diagnosis of SBP is made when there is an elevated ascitic fluid absolute neutrophil count (>250 cells/mm3) without an evident intraabdominal or surgically treatable cause for the infection. SBP most often occurs with portal hypertensive ascites. The most sensitive marker available for such ascites is a serum/ascites albumin gradient >1.0 mg/dL. Traditionally, the empiric treatment for SBP is ampicillin (choice A) plus an aminoglycoside. However, owing to the nephrotoxicity of that regimen in the setting of patients who may require liver transplant, monotherapy with a third-generation cephalosporin is now the treatment of choice. The duration of therapy is 5 days. An alternative regimen is amoxicillin-clavulanic acid.

Azithromycin (choice B) is often used to treat respiratory infections and nongonococcal urethritis and cervicitis. It is not used in SBP.

Penicillin G (choice D) is used to treat syphilis and streptococcal infections. It is not used to treat SBP, which is often caused by a mixture of Gram-positive and Gram-negative bacteria.

Vancomycin (choice E) is used for pseudomembranous colitis and staphylococcal infections. It is not used to treat SBP, which is often due to Gram-negative and -positive bacteria.

 

A 25-year-old man comes to the clinic complaining of an itchy rash for 2-3 weeks on his chest and upper back. He is otherwise healthy and is not taking any medications. On physical examination, there are numerous hypopigmented macules, some coalescing into patches on his chest wall, upper back, and proximal arms bilaterally. There is superficial scaling on some of the lesions, but no erythema or violaceous borders. Wood’s lamp on the lesions revealed yellow-white fluorescence. A scraping of the skin with potassium hydroxide, under the microscope, reveals short hyphae with numerous round spores. The correct diagnosis and treatment for this patient is

Top of Form

  A. erythrasma and topical cleomycin
  B. erythrasma and topical ketoconazole
  C. tinea corporis and topical terbinafine
  D. tinea versicolor and topical cleomycin
  E. tinea versicolor and topical ketoconazole

Bottom of Form

Explanation:

The correct answer is E.Tinea versicolor and topical ketoconazole is the correct answer because this is a common skin disorder caused by Malassezia furfur (also known as Pityrosporum ovale). The disease may occur at any age, but it is much more common during the years of higher sebaceous activity (i.e., adolescence and young adulthood). Some individuals, especially those with oily skin, may be more susceptible. The lesions begin as multiple small, circular macules of various colors (white, pink, or brown) that enlarge radially. The hypopigmented lesions are caused by alterations in melanosome formation and transfer to keratinocytes. The upper trunk is most commonly affected, but it is not unusual for lesions to spread to the upper arms, neck, and abdomen. Diagnosis is confirmed by potassium hydroxide examination of the scale which shows numerous hyphae that tend to break into short, rod-shaped fragments intermixed with round spores in grapelike clusters, giving the so-called “spaghetti and meatballs” pattern. Wood’s light examination shows irregular, pale, yellow-to-white fluorescence that fades with improvement. Treatment includes a a topical selenium sulfide suspension, ketoconazole, zinc pyrithione shampoo, or oral ketoconazole, itraconazole, or fluconazole. A topical antibiotic such as cleomycin (choice D) is the incorrect choice of treatment for these patients.

Erythrasma with topical cleomycin (choice A) and erythrasma with topical ketoconazole (choice B) are both incorrect because erythrasma is caused by Corynebacterium species. Therefore, it should not have any hyphae on scraping. It is a disorder characterized by red to brown scaling patches affecting the intertriginous areas such as the groin, submammary regions, axillae, and toe webs. The correct treatment for erythrasma is either topical erythromycin or cleomycin

Tinea corporis(choice C) is incorrect because tinea corporis lesions, also known as ring worm, tend to be round with elevated erythematous borders that have a central clearing and superficial scales. In addition, tinea corporis is usually caused by the Trichophyton species which do not fluoresce with wood’s lamp examination.

 

  A 28-year-old woman comes to the office because of a 3-hour history of severe nausea, vomiting, abdominal cramps, and diarrhea. She ate lettuce with salad dressing, custard, and pastries and drank stream water at a family picnic at a local park 2 hours before the onset of the symptoms. She is unsure whether anyone got sick. She was “absolutely fine” before she went to the picnic. She does not take any medications. Her temperature is 36.7 C (98.0 F), blood pressure is 110/70 mm Hg, pulse is 65/min, and respirations are 14/min. Physical examination shows mild abdominal tenderness. A stool sample shows large numbers of Gram-positive cocci in clusters. At this time the most correct statement about her condition is:

Top of Form

  A. An antibiotic is indicated to treat her infection
  B. Colonoscopy will show pseudomembranes and friable mucosa
  C. Contaminated stream water at the picnic caused these symptoms
  D. Her symptoms are due to an enterotoxin produced by the organism
  E. It is unlikely that the organism was transmitted from an individual with purulent lesions on the hands, nose, and face

Bottom of Form

Explanation:

The correct answer is D. This patient most likely has staphylococcal food poisoning, which typically presents with nausea, vomiting, abdominal cramps, and diarrhea within hours of consuming the contaminated food. The contaminated foods have usually come into contact with food handlers or contaminated preparation equipment. These food handlers are infected carriers who may have abscesses, boils, and purulent lesions on their bodies. The staphylococcal infection is usually due to an enterotoxin producing Staphylococcus aureus strain. Large amounts of S. aureus are found in the vomitus and the stool of infected patients. The treatment is supportive. Antimicrobials are not indicated.

Even though this patient most likely has staphylococcal food poisoning, an antibiotic is NOT indicated to treat her infection (choice A). This condition is usually self-limited.

Colonoscopy will show pseudomembranes and friable mucosa (choice B) in pseudomembranous colitis, which is usually associated with Clostridium difficile infection. C. difficile typically occurs in patients taking antibiotics and usually presents with fever and large amounts of watery diarrhea.

If this patient had giardiasis, contaminated stream water at the picnic would have caused these symptoms (choice C). However the incubation period for this patient’s symptoms was hours and is therefore more consistent with staphylococcal food poisoning than with giardiasis. Giardiasis typically has an incubation period of days to weeks.

It is unlikely that the organism was transmitted from an individual with purulent lesions on the hands, nose, and face (choice E) is incorrect. This patient most likely has staphylococcal food poisoning, which is often associated with food handlers that have abscesses, boils, and purulent lesions on their bodies.

 

  A 43-year-old man with acquired immunodeficiency syndrome (AIDS) is in the hospital for pneumonia. On his second hospital day, he reports difficulty swallowing his meals. He says that for the last month he has had difficulty swallowing food and medications. He also occasionally feels a burning pain in his upper chest when swallowing. He denies abdominal pain, nausea, or vomiting. Vital signs are: 37.0 C (98.6 F), blood pressure 129/88 mm Hg, pulse 80/min. Examination of his mouth reveals pink oral mucosa and a normal tongue. He has no significant cervical lymphadenopathy. Abdominal examination is normal. The patient’s last CD4 count was performed 5 months ago and at that time was 190/mm3. The most appropriate next step in the management of this patient is to

Top of Form

  A. order esophagogastroduodenoscopy (EGD)
  B. order a Helicobacter pylori antibody test
  C. prescribe a trial of antacids and schedule a follow-up appointment
  D. prescribe oral acyclovir
  E. prescribe oral fluconazole

Bottom of Form

Explanation:

The correct answer is A. The symptoms of dysphagia and odynophagia in a patient with AIDS are highly suspicious for esophagitis. The causes of esophagitis in the setting of AIDS or other immunocompromised states include Candida albicans, cytomegalovirus (CMV), and herpes simplex virus (HSV). The most frequent cause is C. albicans, which accounts for 50-70% of all cases. Esophagogastroduodenoscopy (EGD) is the best way to diagnose the etiology of esophagitis by providing both direct visualization of the esophageal lesions and the ability to obtain biopsies. If the patient has oral thrush and symptoms of esophagitis, the most likely etiology is C. albicans and treatment with fluconazole can be initiated empirically. This patient, however, has normal oral mucosa and needs an EGD prior to treatment.

The Helicobacter pylori antibody test (choice B) is used to determine the etiology of gastric ulcers. This test is not indicated in this patient because he does not have symptoms of ulcer disease. The symptoms of dysphagia and odynophagia in a patient with AIDS are highly suspicious for a fungal or viral esophagitis.

A trial of antacids (choice C) is inappropriate for this patient. The symptoms of dysphagia and odynophagia in a patient with AIDS are highly suspicious for fungal or viral esophagitis. Antacids are used primarily for gastroesophageal reflux disease and will not help in this case. A follow-up appointment is important, however, after the patient has an esophagogastroduodenoscopy (EGD) and is started on appropriate medications.

Treatment with acyclovir (choice D) is premature at this time. There are multiple causes of esophagitis in patients with AIDS and an esophagogastroduodenoscopy (EGD) should be performed to evaluate the esophageal lesions as well as to obtain biopsies. If the esophagitis is caused by herpes simplex virus (HSV), acyclovir is the anti-viral medication of choice.

Treatment with fluconazole (choice E) is premature at this time. There are multiple causes of esophagitis in patients with AIDS and esophagogastroduodenoscopy (EGD) should be performed to evaluate the esophageal lesions as well as to obtain biopsies. If the esophagitis is caused by Candida albicans, fluconazole is the anti-fungal medication of choice.

 

  A 57-year old woman comes to the emergency department because of a “very high fever.” She has diabetes mellitus and hemodialysis-dependent renal failure. She also has hypertension and is status-post total abdominal hysterectomy. She is frail appearing and diaphoretic. Her blood pressure is 170/90 mm Hg and temperature is 38.3 C (101.0 F). Her neck is supple without any specific meningismus. She has a Tesio catheter in her left subclavian vein. Her lungs are clear and she has no costovertebral angle tenderness. Her laboratory studies show a white blood cell count of 23,000/mm3 and a hematocrit of 31%. Her urinalysis is dipstick negative for white blood cells. The most appropriate next step in management is to

Top of Form

  A. begin antibiotic therapy with gentamycin
  B. begin antibiotic therapy with vancomycin and gentamycin
  C. order urinalysis analysis and culture
  D. perform a lumbar puncture and send CSF for analysis and culture
  E. schedule emergent surgical removal of her Tesio catheter

Bottom of Form

Explanation:

The correct answer is B. Infection is the most common cause of death in patients with chronic renal failure. This is followed closely by cardiovascular events. The etiology of increased risk of infection is multifactorial and involves a complex interplay of decreased immune response and complement activation by dialysis membranes all coupled with long-term indwelling components such as catheters. When a dialysis patient presents with infection, the first step in their management is to initiate broad antibiotic coverage based upon the likely causative organisms. This patient has an indwelling catheter and therefore has an increased risk of infection with both coagulase-positive and coagulase-negative Gram-positive cocci. Given the large percentage (25% at most centers) of methicillin resistant Staphylococcus aureus (MRSA), vancomycin is usually initiated until sensitivity data is available. An aminoglycoside is usually added to cover for very common Gram-negative infections.

Begin antibiotic therapy with gentamicin (choice A) is inadequate since the majority of infections in patients such as these will not be covered by an aminoglycoside alone.

Sending her urine for analysis and culture (choice C), although prudent, to perform this with a negative urine dipstick for WBCs, will not change your initial management and decision to cover the patient with broad spectrum antibiotics.

Performing a lumbar puncture and sending a CSF for analysis and culture (choice D) implies meningitis as a cause for the fevers. Meningitis is a rare cause of fevers generally. And although dialysis patients are at mildly increased risk of meningeal infections, in the absence of localized signs and symptoms, the likelihood of meningitis is very low and therefore the risk of an LP is not warranted.

Schedule emergent surgical removal of her Tesio catheter (choice E) is not an appropriate initial management step in a febrile patient. This may be indicated later in the course of care, but concern over removal before antibiotics have begun, is not appropriate.

 

  A 24-year-old man comes to the clinic because of 2 “bumps” on his penis and scrotum. The lesions have been there for approximately 7 months and have been getting progressively larger. They are not painful. He is sexually active with 2 female partners, who are both on oral contraceptive pills and so they do not use barrier contraception. He had chlamydial urethritis last year. His temperature is 37.0 C (98.6 F). Physical examination shows a 3 mm flesh-colored, non-tender, lesion with a “heaped-up” appearance on the shaft of the penis and a 4 mm lesion with a similar appearance on his scrotum. The remainder of the examination is unremarkable. A rapid plasma regain (RPR), VDRL, and fluorescent treponemal antibody absorption (FTA-ABS) test are all nonreactive. In addition to providing the appropriate treatment, he should be told that:

Top of Form

  A. Condoms will prevent the spread of this disease to future sexual partners
  B. His sexual partners should be evaluated because they may be at an increased risk for cervical cancer
  C. Oral suppressive therapy will decrease the frequency of recurrences
  D. The state health department will be contacted because this is a notifiable infectious disease
  E. Treatment will eradicate the infection

Bottom of Form

Explanation:

The correct answer is B. This patient has an anogenital human papillomavirus (HPV) infection (also known as genital warts). These lesions may be confused with condyloma lata (secondary syphilis). However, the “heaped-up” appearance of the flesh colored lesions are more characteristic of HPV and the serology for syphilis is negative. The treatment options for HPV include podophyllin application, cryotherapy, trichloroacetic acid, imiquimod, electrocautery, laser surgery, or surgical excision. This patient needs to tell his sexual partners to be evaluated because HPV infection is associated with cervical cancer.

Condoms may help to decrease the spread of this disease to future sexual partners, but it will not prevent it (choice A) because the treatment does not eradicate the infection from the surrounding tissues. He still should be advised to use condoms.

If this patient had a herpes infection (vesicles and ulcers), oral suppressive therapy would decrease the frequency of recurrences. However, he has an HPV infection, which is not usually treated with oral suppressive therapy (choice C).

It is incorrect to tell him that the state health department will be contacted because this is not a notifiable infectious disease (choice D). Notifiable sexually transmitted diseases are syphilis, chlamydia, gonorrhea, and AIDS.

It is incorrect to tell him that treatment will eradicate the infection (choice E) because while treatment may initially remove the wart, it does not eradicate the HPV from the surrounding tissues.

 

A 37-year-old man comes to the clinic because of painful ulcers on his penis. He has no other symptoms, but he is very concerned because he recently returned from a business trip where he had sexual intercourse with a prostitute and he “obviously” does not want his wife to find out. He “needs” you to do something for him “fast”. He admits to having other sexually transmitted diseases including chlamydia and gonorrhea before he was married. He denies ever having “sores” on his penis or anus in the past. Physical examination shows painful, shallow ulcers on the penis and the perineum. A Tzanck preparation made from a scraping taken from the base of the lesion shows multinucleated giant cells. The results of a tissue culture, which return 5 days later, show herpes simplex virus-2. You prescribe a 10-day course of oral acyclovir. He should be told that:

Top of Form

  A. Chronic daily suppressive therapy with acyclovir does not reduce the frequency of reactivation
  B. He needs to use condoms with his wife only when lesions are present
  C. Herpes simplex virus-2 usually becomes a life-long latent infection in the epithelial cells
  D. The recurrence rates for herpes simplex virus-1 are higher than that of herpes simplex virus-2
  E. Vaccination against the hepatitis B virus is recommended at this time

Bottom of Form

Explanation:

The correct answer is E. This patient has herpes simplex virus-2, which is a very common sexually transmitted disease in the United States. It is estimated that 22% of the adult population has antibodies to the virus, even though only a small percentage of this group ever reports symptoms. Unfortunately, approximately 90% of those who report symptoms will have a recurrence. Transmission is possible during asymptomatic periods and subclinical shedding of the virus is well documented. Herpes simplex virus-2 can be transmitted by skin-to-skin contact when lesions are present, even when a condom is used. Acyclovir, an antiviral agent that inhibits viral DNA polymerase, shortens the duration of symptoms and the viral excretion time. Individuals who have recently acquired a sexually transmitted disease that have not been vaccinated against the Hepatitis B virus should be offered the Hepatitis B vaccine.

Many patients with frequent reactivation of herpes simplex virus-2 require chronic daily suppressive therapy with acyclovir to reduce the frequency of reactivation. It is incorrect to say that chronic daily suppressive therapy with acyclovir does not reduce the frequency of reactivation (choice A).

Transmission of HSV-2 is possible even during asymptomatic periods and subclinical shedding of the virus is well documented. Therefore he needs to use barrier protection all of the time, even when lesions are not present, to decrease the likelihood of transmission. He needs to use condoms with his wife only when lesions are present (choice B) is incorrect

Herpes simplex virus-2 usually becomes a life-long latent infection in the sensory ganglion cells, not epithelial cells (choice C). Once stressors such as trauma, fever, UV light, sexual intercourse, or immunodeficiency reactivate the virus in the neural tissue, HSV is transported back to the epithelium and another episode occurs.

The recurrence rate for herpes simplex virus-1are approximately 55% and 90% for herpes simplex virus-2. Therefore, recurrence rates for herpes simplex virus-1 are higher than that of herpes simplex virus-2 (choice D) is incorrect.

 

A 29-year-old man comes to the office because of a recurrent rash that worsens in the summer season and recurs more frequently with humid weather. Usually the rash involves his upper back and occasion spreads to the shoulders and proximal arms. It is occasionally pruritic. Physical examination shows multiple hypopigmented oval macules and patches on his upper back. His hands and feet are not involved and all nails appear normal. You examine scrapings of the scale with potassium hydroxide under the microscope and find numerous short hyphae with multiple round spores in clumps. You should tell the patient that he has

Top of Form

  A. tinea corporis and should be treated with oral terbinafine
  B. tinea corporis and should be treated with topical ketoconazole
  C. tinea corporis and should be treated with topical terbinafine
  D. tinea versicolor and should be treated with oral terbinafine
  E. tinea versicolor and should use ketoconazole shampoo

Bottom of Form

Explanation:

The correct answer is E. This patient has tinea versicolor and he should be treated with ketoconazole shampoo. Tinea versicolor, also known as pityriasis versicolor, usually presents on the upper trunk and extends onto the upper arms, finely scaling, guttate, or nummular patches appear, particularly on young adults who perspire freely. The individual patches are yellowish or brownish macules in pale skin or hypopigmented macules in dark skin, with delicate scaling. Mild itching and inflammation may be present. This common disease is most prevalent in the tropics, where there is high humidity, high temperatures and frequent exposure to sunlight. The fungus is easily demonstrated in scrapings of the scales. Microscopically, there are short, thick fungal hyphae and large numbers of variously sized spores. This combination is commonly referred to as “spaghetti and meatballs”. Treatment is accomplished with topical imidazoles, oral ketoconazole, or itraconazole. Oral terbinafine (choice D) has been shown to be ineffective.

Tinea corporis (choices A, B, C) are all incorrect, because the lesions described on this young man are not consistent with tinea corporis. Typical tinea corporis lesions (also known as ring worm) are round, erythematous borders with central clearing and superficial scales. The borders tend to be elevated rather than flat.

 

  A 37-year-old woman with a history of intravenous drug use, hepatitis B, asthma, and acquired immunodeficiency syndrome (AIDS) is admitted to the hospital because of fever, night sweats, and malaise. Her last CD4 count was 1 month ago and measured 180/mm3. Vital signs are: temperature 38.5 C (101.3 F), blood pressure 145/76 mm Hg, and pulse 90/min. Physical examination is significant for a soft diastolic murmur heard best at the lower left sternal border. Auscultation of the lungs reveals diffuse rhonchi. The abdominal and neurologic exams are unremarkable. The next step in managing this patient is

Top of Form

  A. analysis and culture of spinal fluid
  B. a blood culture
  C. a CT of the head
  D. a urinalysis
  E. an x-ray of the chest
  F. an x-ray of the abdomen

Bottom of Form

Explanation:

The correct answer is E. A chest x-ray is essential because of the rhonchi heard on lung exam. A patient with AIDS is at risk for an opportunistic infection and the risk increases greatly as the CD4 lymphocyte count drops. An echocardiogram should also be obtained because of the cardiac murmur which is suspicious for endocarditis. Intravenous drug users are at particular risk for right-sided endocarditis. Statistically, however, community acquired pneumonia is still more probable than endocarditis even with these clinical findings.

The patient has no mental status changes, neurological deficits, or symptoms of headache to suggest meningitis as the cause of fever. Hence, an analysis and culture of spinal fluid (choice A) is not necessary at this time. A chest x-ray is essential because of the rhonchi heard during the lung exam. A patient with AIDS is at risk for opportunistic infections and the risk increases greatly as the CD4 lymphocyte count drops. An echocardiogram must also be obtained because of the cardiac murmur which is suspicious for endocarditis. Intravenous drug users are at particular risk for right-sided endocarditis.

Obtaining blood cultures (choice B) is not the most appropriate next step. A chest x-ray is essential because of the rhonchi heard on lung exam. If the suspicion for endocarditis is still high after the chest x-ray, blood cultures should be obtained.

A CT of the head (choice C) is not indicated because the patient does not have mental status changes, neurologic deficits, or symptoms of headache. A chest x-ray is essential because of the rhonchi heard on lung exam.

A urinalysis (choice D) is not necessary at this time because the patient does not have urinary symptoms such as dysuria or increased frequency.

An x-ray of the abdomen (choice F) is not necessary at this time because the patient has a normal abdomen on examination and no abdominal symptoms.

 

  A 64-year-old woman comes to the emergency department because of a non- healing ulcer in the right foot. She is a known insulin-dependent diabetic, is hypertensive, and is on dialysis for chronic renal failure. She noticed an ulcer in the plantar aspect of her foot a week ago following a hike on marshy grounds. She started taking oral antibiotics that she had at home from a past illness. However, since yesterday, she has noticed a foul smelling discharge from the ulcer, along with foot swelling and fever. Her temperature is 39.1 C (102.4 F), blood pressure 140/68 mm Hg, and pulse 88/min. Local examination of the right lower extremity shows swelling of the lower leg and foot with crepitus. A 3×2 cm. ulcer at the base of the great toe is noticed on the plantar aspect with foul smelling serous brownish discharge. Lower extremity pulses are diminished bilaterally. Laboratory studies show:

The most appropriate next step in management to limit disability is to

Top of Form

  A. administer insulin and send her home with oral antibiotics
  B. order foot and leg x-rays
  C. perform incision and drainage of the ulcerated area
  D. provide intravenous antibiotics
  E. repeat serum potassium level

Bottom of Form

Explanation:

The correct answer is B. Diabetic foot ulcers, when inadequately treated, can result in wet gangrene and necrotizing infections. Serous brownish discharge, swelling of foot and leg with crepitus, fever, and high leukocyte counts, are hallmarks of a necrotizing infection in a diabetic foot. Immediate foot and leg x-rays are essential to diagnose subcutaneous air and to delineate the level of spread. Following this, the patient should be taken to the operating room for a wide debridement or even an amputation depending upon the extent of necrosis of the tissues.

Control of blood sugar is essential for proper wound healing in a diabetic, but the removal of the source of infection is important in preventing further complications of infection (choice A). In this patient, administering insulin and antibiotics are essential, but surgical debridement is more important in limiting the disability. She requires further evaluation and treatment and should not be sent home.

Simple incision and drainage (choice C) is not adequate in treating a necrotizing infection.

This patient is already taking antibiotics at home. Although she needs intravenous antibiotics (choice D) for better control of the infection, surgical debridement of the foot is the first essential step to control the damage and limit disability.

High serum potassium level is a consequence of her chronic renal failure and increased cellular lysis in this necrotizing infection. Without controlling further spread of infection, serum potassium level is bound to increase and can cause cardiac dysrhythmias. To limit the disability, removal of the source of high potassium level is essential rather than repeating potassium level (choice E).

 

  A 64-year-old woman comes to the emergency department because of a non- healing ulcer in the right foot. She is a known insulin-dependent diabetic, is hypertensive, and is on dialysis for chronic renal failure. She noticed an ulcer in the plantar aspect of her foot a week ago following a hike on marshy grounds. She started taking oral antibiotics that she had at home from a past illness. However, since yesterday, she has noticed a foul smelling discharge from the ulcer, along with foot swelling and fever. Her temperature is 39.1 C (102.4 F), blood pressure 140/68 mm Hg, and pulse 88/min. Local examination of the right lower extremity shows swelling of the lower leg and foot with crepitus. A 3×2 cm. ulcer at the base of the great toe is noticed on the plantar aspect with foul smelling serous brownish discharge. Lower extremity pulses are diminished bilaterally. Laboratory studies show:

The most appropriate next step in management to limit disability is to

Top of Form

  A. administer insulin and send her home with oral antibiotics
  B. order foot and leg x-rays
  C. perform incision and drainage of the ulcerated area
  D. provide intravenous antibiotics
  E. repeat serum potassium level

Bottom of Form

Explanation:

The correct answer is B. Diabetic foot ulcers, when inadequately treated, can result in wet gangrene and necrotizing infections. Serous brownish discharge, swelling of foot and leg with crepitus, fever, and high leukocyte counts, are hallmarks of a necrotizing infection in a diabetic foot. Immediate foot and leg x-rays are essential to diagnose subcutaneous air and to delineate the level of spread. Following this, the patient should be taken to the operating room for a wide debridement or even an amputation depending upon the extent of necrosis of the tissues.

Control of blood sugar is essential for proper wound healing in a diabetic, but the removal of the source of infection is important in preventing further complications of infection (choice A). In this patient, administering insulin and antibiotics are essential, but surgical debridement is more important in limiting the disability. She requires further evaluation and treatment and should not be sent home.

Simple incision and drainage (choice C) is not adequate in treating a necrotizing infection.

This patient is already taking antibiotics at home. Although she needs intravenous antibiotics (choice D) for better control of the infection, surgical debridement of the foot is the first essential step to control the damage and limit disability.

High serum potassium level is a consequence of her chronic renal failure and increased cellular lysis in this necrotizing infection. Without controlling further spread of infection, serum potassium level is bound to increase and can cause cardiac dysrhythmias. To limit the disability, removal of the source of high potassium level is essential rather than repeating potassium level (choice E).

  A 2-week-old previously healthy baby boy who was born full term without any complications is brought to the emergency department by his mother for a fever. The baby appears well, but his temperature is found to be 38.9 C (102.0 F). After the appropriate blood work and cultures are done, you decide to start this baby on antibiotics. The most appropriate pharmacotherapy at this time is

Top of Form

  A. ampicillin and cefazolin
  B. ampicillin and cefotaxime
  C. ceftriaxone alone
  D. vancomycin and cefotaxime
  E. vancomycin and cefuroxime

Bottom of Form

Explanation:

The correct answer is B. The organisms most responsible for bacteremia in a baby less than 4 weeks of age are Group B Streptococcus, E. coli, and Listeria; i.e., vaginal flora. Other less common organisms are anaerobes, Pneumococcus, and other Gram-negative enterics. Therefore, the antibiotics of choice are ampicillin (to cover for Listeria and other Gram-positive organisms) and a third-generation cephalosporin. Traditionally, cefotaxime is the third generation cephalosporin of choice.

A first-generation cephalosporin, such as cefazolin, in addition to ampicillin (choice A) would give good Gram-positive coverage, but would not effectively cover Gram-negative organisms. In a baby this age, you have to worry about infections with vaginal organisms such as E. coli and other Gram-negatives, therefore a third-generation cephalosporin or an aminoglycoside is a more appropriate choice as an addition to the ampicillin.

Ceftriaxone (choice C) is a third-generation cephalosporin with very good central nervous system penetration. Unfortunately, ceftriaxone increases the risk for hyperbilirubinemia and does not cover Listeria sufficiently. Therefore, it is not the treatment of choice for a 2-week-old with possible sepsis.

Although vancomycin and cefotaxime (choice D) would cover the appropriate organisms, vancomycin would be “overkill” in this previously healthy baby with no history of any in-dwelling lines or hardware. Also, there are certain strains of Listeria that are not susceptible to vancomycin. If this baby had a history of prematurity, had a prolonged hospital stay and therefore at increased risk of nosocomial infection, or had any in-dwelling catheter that put him at increased risk for Staphylococcus aureus, then vancomycin would be an appropriate addition to the regimen. As you are given a history of a healthy baby, ampicillin and cefotaxime are the appropriate regimen while awaiting culture growth and organism sensitivity.

Vancomycin and cefuroxime (choice E) is also not an appropriate choice. As above, vancomycin is not indicated nor sufficient in this patient and unnecessarily exposes this child to side effects such as red man’s syndrome and ototoxicity. Cefuroxime is not a good choice because it is a second-generation cephalosporin, and therefore does not cover the Gram-negative organisms such as E. coli as well as a third-generation cephalosporin like cefotaxime would.

 

 

 

A 38-year-old man comes to the clinic complaining of a 2-week history of an itchy, scaly, red rash on his left upper chest. He initially treated it with an over-the-counter antibiotic cream. After 5 days with no apparent improvement, he switched to a hydrocortisone cream, which he has been using for the past 9 days. This decreased the redness and itch, but since switching to the steroid, the area of involvement has been growing steadily. He is a school bus driver and he enjoys gardening. He says that he was spending excessive time outside in the sun working in his garden for several weeks leading up to the appearance of the rash. He has been told he has “borderline” diabetes and is currently on a 6-month trial of dietary modification and exercise to see if he can avoid oral hypoglycemic agents. Physical examination reveals an overweight Caucasian man with a scaly, slightly erythematous, 6 by 10 cm plaque which is slightly indurated at the periphery. No other significant lesions are seen. The most appropriate next diagnostic step is to

Top of Form

  A. check an antinuclear antibody titer
  B. check the patient’s thyroid stimulating hormone
  C. obtain a skin biopsy
  D. perform a KOH preparation of the scale
  E. test the patient for the presence HIV antibody

Bottom of Form

Explanation:

The correct answer is D. This is an excellent history and physical exam for tinea corporis, a fungal infection of the skin. A scaly, erythematous plaque which is accentuated or indurated at the periphery, must always be checked for fungal hyphae with a KOH preparation. The dermatologic adage, “if there is scale, scrape it” [to check for fungus] is very sound advice. The application of topical steroids to tinea corporis will generally help decrease the erythema, but will allow the fungus to grow unchecked and the involved area to enlarge. A scaly area that is worsened by topical steroids should be considered tinea until proven otherwise. Tinea in adults is generally acquired by contact with children, pets, or soil. Tinea is also much more common in patients with diabetes.

Checking an antinuclear antibody titer (choice A) can be useful if one is concerned about autoimmune or rheumatologic disorders such as lupus erythematosus. Several of the criteria for the diagnosis of lupus erythematosus are dermatologic manifestations including the presence of a malar rash, discoid lesions, and oral ulcerations. Sun exposure is a known trigger and will exacerbate the cutaneous manifestations of lupus erythematosus. Discoid lesions of lupus erythematosus (clinically appearing as circular, atrophic, hypopigmented plaques occasionally with scale) can sometimes be confused with fungal infections, but are generally confined to the head and neck. Discoid lupus would also be helped by topical steroids, not exacerbated by them.

Checking a TSH level (choice B) is a sensitive and specific way for detecting hypo- and hyperthyroidism. Weight gain can be a sign of hypothyroidism; however the vast majority of obese individuals are euthyroid. The dermatologic manifestations of hypothyroidism include rough and dry skin, coarse hair, and brittle nails. Scaly plaques are not a sign of thyroid disease.

A skin biopsy (choice C) is an excellent method to diagnose skin disorders and if diagnostic dilemmas occur, it can be used to differentiate between various rashes. Before you put the patient through the discomfort, scarring, and expense of a skin biopsy however, tests you can perform and evaluate right in your office, such as a KOH preparation, are preferable.

Checking for the presence of HIV antibody (choice E) may be indicated in patients who present with widespread fungal infections without any known predisposing factor such as diabetes mellitus or other immunocompromising condition.

 

A 4-year-old boy is brought to the office because he has refused to walk for the past 24 hours. He was well until yesterday afternoon when he woke from a nap complaining of feeling “wobbly”. When his parents got him up to walk he was extremely unsteady and they needed to hold him to keep him from falling over. He has not improved at all over the last day. His unsteadiness persists when he is sitting down. His temperature is 37.0 C (98.6 F), pulse is 100/min, and respirations are 24/min. He has horizontal nystagmus, which is worse at the extremes of gaze. He appears markedly ataxic and his gait is broad based. He has no papilledema. If obtained, the information that would be most pertinent to his current condition is

Top of Form

  A. concomitant diarrhea
  B. a cousin with a brain tumor
  C. a history of varicella infection 3 weeks ago
  D. previous history of epilepsy in this patient
  E. recent streptococcal pharyngitis

Bottom of Form

Explanation:

The correct answer is C. The history given above is a classic presentation of acute post-infectious cerebellar ataxia. 25% of children with this disorder have a history of varicella infection within 1 month prior to the onset of the disorder and 5% of children have a previous history of varicella vaccination. The onset of acute cerebellar ataxia is usually explosive, often with the child awakening from sleep with maximal symptoms of ataxia and nystagmus. In any child with acute ataxia, it is important to obtain a drug screen and a careful history of any possible toxic exposures since poisoning and acute cerebellar ataxia account for most of the cases of acute ataxia in children. Resolution of the symptoms occurs in most children over a period of weeks to months.

Concomitant diarrhea (choice A) is not relevant to this presentation. Although there are forms of Guillain-Barre syndrome that have ataxia as a prominent component, this child does not have the other symptoms associated with these variants of GBS (ophthalmoplegia, depressed reflexes), and Campylobacter diarrhea usually precedes the onset of GBS.

While an intracranial tumor is always of concern in a child with neurologic symptoms, the history above is atypical for tumors, which usually present more indolently. The history of a cousin with a brain tumor does not increase the likelihood that this child’s symptoms are due to an intracranial mass (choice B).

Pseudoataxia (choice D) is a syndrome of recurrent bouts of ataxia that are actually atypical seizures on EEG. The attacks are similar in timing to seizures and there may be a postictal state. They are not likely to persist over days as in this child.

Recent streptococcal pharyngitis (choice E) is of importance in movement disorders associated with acute rheumatic fever, such as Sydenham chorea, but does not predispose to any known form of ataxia.

 

 

You are called to see a 41-year-old man, who is 6 days post—transplant from an allogenic bone marrow transplant, for a fever spike to 39.8 C (103.6 F). He was diagnosed with acute myelogenous leukemia seven weeks prior. He underwent induction with busulfan and cyclophosphamide and received an HLA-matched graft from his sister. His current medications include trimethoprim-sulfamethoxazole three times weekly, levofloxacin three times weekly, cyclosporine daily, and a beta blocker for his hypertension. His most recent white cell count was 1300 cells/mm3 with an absolute neutrophil count of less than 100. The patient has no indwelling central vascular catheters of any type. The most appropriate antibiotic for this patient is

Top of Form

  A. cefotetan, intravenously
  B. ceftazidime, intravenously
  C. imipenem/cilastatin, intravenously
  D. piperacillin, intravenously
  E. vancomycin, intravenously

Bottom of Form

Explanation:

The correct answer is B. This is an example of a patient with fever and neutropenia. This is a common scenario encountered on ward services. Neutropenia is defined as a count less than 500/microliter, and fever is defined as a single oral temperature greater than 38.5 C (101.3 F). Standard therapy is empiric and consists of monotherapy with an antipseudomonal third-generation cephalosporin or of an antipseudomonal penicillin and an aminoglycoside.

Cefotetan (choice A) is a second-generation cephalosporin often used to treat bowel infections. It has no activity against pseudomonas.

Imipenem/cilastatin (choice C) is a combination of a cell wall synthesis inhibitor (Imipenem) and a beta lactamase inhibitor (cilastatin) that is very potent and efficacious. However, like piperacillin, its broad use will encourage resistance and is therefore not routinely used.

Piperacillin (choice D) is a new generation penicillin with very broad coverage of both Gram-positive and Gram-negative organisms including pseudomonas. It is however not generally indicated because of concerns about developing resistance.

Vancomycin (choice E) is used in febrile and neutropenic patients in addition to ceftazidime when the patient has an indwelling central vascular catheter.

 

You are called to the emergency department to evaluate a 6-year-old girl who has developed a rash on her distal extremities 2 days ago that has been progressing toward her trunk. She has had a fever and arthralgias over the past 2 days for which she was given acetaminophen. On further questioning, the patient’s mother reports that the patient was bitten by her pet rat a few days prior to onset of the fever and rash; however, the site appears to be healing well. Her mother reports that the girl has a normal past medical history without any significant health problems. Laboratory studies show leukocytosis with an elevated neutrophil count. Blood culture results are pending. At this time the most correct statement about this patient’s condition is:

Top of Form

  A. Her rash is classic for Rocky Mountain Spotted Fever and she should be started on antibiotics for rickettsial organisms
  B. Meningitis can be ruled out because the rash usually starts on the trunk and extends peripherally
  C. The patient should be admitted and started on broad-spectrum antibiotics to cover Streptobacillus moniliformis and Spirillum minor
  D. This is a drug eruption secondary to acetaminophen and you should advise the patient’s mother to stop acetaminophen and give ibuprofen instead
  E. This is most likely a viral exanthem and will resolve spontaneously in 1-2 weeks

Bottom of Form

Explanation:

The correct answer is C. Rat bite fever is an acute febrile illness that is usually accompanied by a skin rash. It is caused by either Streptobacillus moniliformis or Spirillum minor which are the bacteria that generally infect humans as a result of the bite of a rat, mouse, or other rodents. Following a rat bite, there is minimal local inflammation with prompt healing of the wound. If the infection is not halted, bacteremia ensues and lesions distant from the bite appear 1-3 days after the bite. Manifestations include fever, rigors, headaches, malaise, and arthritis. Treatment of choice is penicillin G or tetracycline for patients allergic to penicillins. Occasionally the course is complicated by endocarditis, meningitis, myositis, abscesses, splenic or renal infarction, brain abscess, and sepsis.

Rocky mountain spotted fever (choice A) is incorrect because there is no history of tick bite reported. In addition, in a child of this age group, fever accompanied by an extremity rash should alert physicians to treat and cover meningitis organisms until blood culture results become available. Therefore, since meningitis cannot be definitively ruled out at this time, (choice B) is also incorrect.

Drug eruption (choice D) is incorrect for two reasons. First, drug eruptions typically start on the trunk and extend peripherally. Second, fever and arthritis usually do not accompany typical drug eruptions.

Viral exanthem (choice E) is incorrect at this stage of evaluation. One should rule out more urgent causes of fever and rash before concluding that it is viral exanthem.

 

  A 66-year-old woman comes to the clinic complaining of severe pain across her chest and abdomen. You treated the patient for shingles 5 months ago and at that time she had a shingles band at the right T8 level. The current pain is in the same region where she had her shingles. She states that she cannot stand to have her clothes touch the area and that even shower water hurts. She has hypertension and glaucoma for which she takes beta blocker eye drops and lisinopril. At the level of her T8 dermatome on the right, she has marked allodynia, primary and secondary hyperalgesia. The area is exceptionally tender to palpation. The most appropriate therapy is at this time is

Top of Form

  A. acyclovir
  B. amantadine
  C. amitriptyline
  D. lidocaine cream
  E. oxycodone, sustained release

Bottom of Form

Explanation:

The correct answer is C. This patient has established postherpetic neuralgia. She has all of the classical signs of neuropathic pain including, allodynia (nonpainful stimuli eliciting pain), primary hyperalgesia (pain increasing in intensity with stimulation), and secondary hyperalgesia (surrounding tissue having pain). This entity often follows the varicella zoster reactivation disease shingles and it follows the distribution of the original infection. The main risk factor for the neuralgia is increasing age. There is excellent clinical data showing dramatic pain relief from the use of tricyclic antidepressants. The mechanism of action is unknown.

The use of acyclovir (choice A) is indicated for the prevention of postherpetic neuralgia and hastening the resolution of the zoster infection. These anti-retroviral agents have no efficacy in pain control for established postherpetic pain.

There is no benefit to the use of amantadine (choice B). This class of drugs has benefit in decreasing infectivity from influenza virus by inhibiting early stages of the infection process.

There is some evidence that topical lidocaine cream (choice D) offers some pain relief from neuropathic pain, but compared to tricyclic therapy, the benefits are minimal.

Narcotic drugs such as oxycodone (choice E) are actually very ineffective at controlling neuropathic pain. In fact, they have no recommended role in the management of this type of pain. Oxycodone, sustained release, is an oral sustained release formulation of morphine and is used for the treatment of chronic, nonneuropathic pain conditions. Sustained release formulation are not to be used for the management of acute pain.

 

  A 29-year-old man comes to the office because one of his 3 sexual partners recently had a Pap smear that showed dysplasia and koilocytic changes. Her physician recommended that all of her sexual partners be evaluated. He has always been healthy and has never had any sexually transmitted diseases. All of his partners are “on the pill” so they do not use condoms. Physical examination is completely unremarkable. There are no visible lesions on his anogenital region. He is still very concerned that he has an infection that you cannot see. The most appropriate next step is to

Top of Form

  A. advise him to return if he develops any lesions
  B. apply vinegar to his penis and scrotum
  C. recommend that he use condoms during all sexual activity
  D. send for a fluorescent treponemal antibody absorption (FTA-ABS) serology
  E. take random biopsies of the penis
  F. tell him that he is healthy

Bottom of Form

Explanation:

The correct answer is B. This patient’s girlfriend most likely has human papillomavirus (HPV) infection, which is associated with dysplastic changes and cervical cancer. This patient should be evaluated for an HPV infection, and if there are no visible lesions, acetic acid (vinegar) should be applied to the anogenital region to detect the presence of the virus. Invisible lesions typically turn white when acetic acid is applied. This is thought to occur because the acetic acid causes maceration and swelling of virally induced epithelial hyperplasia, which usually has an increased glycogen content and enhanced permeability. Even though this is not specific for HPV and false-positives can occur, it may enhance the detection of an HPV infection.

If no lesions are found when acetic acid is applied, you should advise him to return if he develops any lesions (choice A) and recommend that he use condoms during all sexual activity (choice C). Condoms will probably not completely prevent the spread of infection, but they should theoretically reduce transmission.

Since his sexual partner most likely has an HPV infection and he is sexually active with many partners, syphilis screening may be appropriate, but the VDRL (Venereal Disease Research Laboratory) and RPR (rapid plasma reagin) tests are used for screening, not the FTA-ABS (choice D). The FTA-ABS is more specific, but it is usually not considered a screening test because it is more expensive and remains reactive in patients with a prior, treated syphilis infection.

Taking random biopsies of the penis (choice E) is completely inappropriate, and it will make a patient very unhappy. Acetic acid should be applied to help see invisible lesions and biopsies can be taken from suspicious areas.

Since many patients infected with HPV have no visible signs and symptoms, it is inappropriate to tell him that he is healthy (choice F) before further evaluation (application of acetic acid).

  A 13-year-old boy comes to the office because of an 8-day history of a fever, sore throat, and extreme “tiredness”. He also complains of a “red rash” that he noticed 5 days ago. He had been seen at a 24-hour clinic 6 days ago for a sore throat, and even though no diagnostic test was performed, he was treated with a 5-day course of ampicillin. He is usually very healthy and has never had “strep throat” before. His temperature is 37.8 C (100.0 F). Physical examination shows cervical adenopathy, tonsillar enlargement with a pharyngeal exudate, and a macular rash on his trunk. Laboratory studies show an elevated white blood cell count with 40% atypical lymphocytes. The most appropriate next step is to

Top of Form

  A. obtain a rapid strep test
  B. obtain a throat culture
  C. order a heterophile test
  D. order a Tzanck smear of a scraping from the rash
  E. prescribe a 7-day course of prednisone

Bottom of Form

Explanation:

The correct answer is C. This patient most likely has infectious mononucleosis (IM), which is a disease caused by the Epstein-Barr virus. The pharyngitis, fever, lymphadenopathy, fatigue, and malaise are common findings early on in the disease. After about 2 weeks, splenomegaly may be a prominent feature. The heterophile test is used to diagnosis the infection. It is usually a self-limited infection that only requires bed rest and analgesics. Physical activity should be avoided for a month to avoid the risk of splenic rupture. If it does occur, the symptoms may be abdominal pain referred to the shoulder and shock. Infectious mononucleosis is often associated with thrombocytopenia and a morbilliform or papular rash. Complications of IM include splenic rupture, airway obstruction from hypertrophied tonsils and adenoids, encephalitis, meningitis, hepatitis, interstitial nephritis, myocarditis, and pericarditis.

A rapid strep test (choice A) is 95% specific for streptococcal pharyngitis when it is positive, however it is not cost effective in this case because the laboratory studies show atypical lymphocytes, which are often seen in infectious mononucleosis, not in streptococcal infections. A heterophile test is used to diagnose IM.

A throat culture (choice B) is often used to diagnose streptococcal pharyngitis when the rapid strep test is negative and the clinical suspicion is strong.

A Tzanck smear of a scraping from the rash (choice D) is unnecessary because this patient most likely has infectious mononucleosis, not chicken pox. A Tzanck smear of a vesicle from a patient with chicken pox (varicella-zoster virus) will show multinucleated giant cells. The rash of chicken pox is often described as lesions in various stages of healing (maculopapular, vesicular, crusting, etc.). This patient’s rash is most likely due to the ampicillin. The mechanism is unknown, but patients with infectious mononucleosis that are treated with ampicillin develop a macular rash.

A 7-day course of prednisone (choice E) is inappropriate at this time in this patient with infectious mononucleosis (IM). Steroids are sometimes necessary in patients with IM who have severe thrombocytopenia and to prevent airway obstruction when severe tonsillar hypertrophy is present. It is not recommended in uncomplicated IM because it may increase the chance for the development of a superinfection with a bacteria.

 

  A 32-year-old HIV-positive intravenous drug abuser is admitted to the hospital following a seizure. He was diagnosed as HIV positive a few years ago, and is currently taking antiviral therapy. He has no previous history of seizures or any other medical problems. He is awake, alert, and oriented. Neurological examination is normal. A CT scan of the brain performed with intravenous contrast shows 3 ring-enhancing lesions. The patient is started on anticonvulsant medications. The most appropriate next step in management is to

Top of Form

  A. obtain CT scan of chest to rule out lymphoma
  B. obtain viral titers
  C. schedule a brain biopsy
  D. start empiric treatment for toxoplasmosis
  E. start empiric treatment for tuberculosis

Bottom of Form

Explanation:

The correct answer is D. HIV patients with focal brain lesions are treated with 2-3 weeks of empiric treatment for toxoplasmosis followed by biopsy (choice C) if the radiological or clinical condition deteriorates. Toxoplasma gondii, the protozoan that causes toxoplasmosis, accounts for 50-70% of focal brain lesions in these patients. 10-20% of focal lesions are CNS lymphomas.

Primary CNS lymphoma (choice A) is a rare intracranial tumor, accounting for 1.5% of all primary brain tumors. CNS lymphoma is still significantly more common in HIV-positive patients, even compared to that in generally immunosuppressed populations. When empiric therapy for toxoplasmosis fails, CNS lymphoma should be ruled out.

Viral titers are of no value in this patient (choice B). Although opportunistic infections such as progressive multifocal leukoencephalopathy caused by papova virus JC, Cytomegalovirus, herpes, toxoplasmosis, and cryptococcosis can cause HIV-related CNS diseases, incidence is low and not of primary etiology.

Tuberculosis causing CNS symptoms is not common (choice E) in this patient population.

 

  A 22-year-old woman is brought to the emergency department by ambulance. She is accompanied by her roommate who states that the patient developed a fever and some confusion 3 hours before, and approximately 30 minutes ago became unconscious. The roommate reports that the patient was complaining of a stiff neck and headache a few hours before she became ill. The roommate knows of no significant medical history but reports that the patient is a volunteer at a local children’s hospital. Initial examination shows the patient to be non-responsive. Her temperature is 40.4 C (104.7 F), blood pressure is 70/40 mm Hg, pulse is 140/min, and respirations are 32/min. There are diffuse petechial and purpuric lesions across the hands, face, and arms. After tracheal intubation, infusion of pressors fails to augment the blood pressure and it remains at 65/35 mm Hg. The most appropriate next step in this patient’s care is to

Top of Form

  A. begin heparin therapy only
  B. give her high-dose corticosteroids, intravenously
  C. initiate chest compressions
  D. perform a lumbar puncture
  E. transfer her to the intensive care unit

Bottom of Form

Explanation:

The correct answer is B. This patient almost certainly has meningococcal sepsis and Waterhouse-Friderichsen syndrome (meningococcemia and adrenal hemorrhage). These patients require exogenous steroids for acute adrenal insufficiency, although they have not been shown to alter survival outcomes. This is presumably due to the fact that these patients are critically ill and are at grave risk for renal and pulmonary failure, massive coagulopathy, and irreversible septic shock. She requires high doses of intravenous corticosteroids and saline and antibiotics.

A characteristic of all sepsis syndromes is disseminated intravascular coagulation (DIC), a consumptive coagulopathy. This patient also has microangiopathic hemolysis secondary to massive small vessel clotting. The correction of her coagulation disorder requires coagulation factors and resolution of her sepsis. Heparin (choice A) may be necessary, but it should only be used in combination with cryoprecipitate and platelets because it may cause bleeding if given alone.

There is no need to initiate chest compressions (choice C) since the patient has a blood pressure, albeit a very weak one. She is in profound septic shock with concomitant adrenal insufficiency and requires catecholamines and steroids.

There is no need for a lumbar puncture (choice D) since this patient has disseminated disease and blood cultures will certainly reveal the presence of the organism.

It is appropriate to transfer the patient to an intensive care unit (choice E) once her condition is stabilized. Patients such as this must be stabilized before being moved since transport through the hospital exposes the patient to a situation where further decompensation is possible while in transit.

 

  A 25-year-old athletic appearing man comes to the office with a recurrent rash in his groin. He denies any significant medical history. He recalls that a similar rash recurs each summer with associated mild pruritus. It usually improves significantly in the winter and he has not been treated. He normally wears tight jockey shorts. Physical examination shows a sharply marginated, scaly, red eruption on the inner thighs that spares the testicles. No satellite lesions are appreciated. The penoscrotal fold and the scrotal sac do not appear to be involved. He also has hypertrophic, discolored, thickened toenails involving only the first toe of both feet. The most appropriate management is to

Top of Form

  A. prescribe a topical erythromycin cream
  B. recommend a topical steroid cream
  C. recommend a topical anti-candidal treatment
  D. recommend a topical antifungal agent
  E. tell him that no treatment is indicated for this anxiety induced dermatosis

Bottom of Form

Explanation:

The correct answer is D. Topical antifungal treatment is correct, because tinea cruris occurs most frequently in men on the upper and inner surfaces of the thighs, especially during the hot summer, if the humidity is high. It begins as a small erythematous, scaling or vesicular and crusted patch, that spreads peripherally and partly clears in the center. In contrast to candida infection, the borders are well demarcated without satellite lesions. Heat and humidity are the predisposing factors for the development of tinea cruris. Tight jockey shorts, which prevent evaporation of the increased perspiration produced during warm weather, may be an additional predisposing factor. Treatment of tinea cruris usually includes methods to reduce perspiration, keeping the area as dry as possible with antifungal powder. Additional topical or oral antifungal agents may be necessary for some patients.

Topical erythromycin (choice A) is incorrect because this is used for treating erythrasma. The crural region is not only a common site for tinea cruris, but also for bacterial and candidal infections. Erythrasma is caused by Cornynebacterium minutissimum. and typically treated with topical erythromycin or clindamycin.

Topical steroids (choice B) is incorrect because fungus thrive in steroid-rich environments and this can worsen the infection.

Topical anti-candidal treatment (choice C) is incorrect because, as stated above, candidal infections usually have satellite lesions, and the color the lesions are generally bright red.

Anxiety induced dermatosis (choice E) is incorrect because this patient’s history and exam point to a rash that’s associated with heat and humidity, and he has associated onychomycosis on his toenails.

 

A 23-year-old college student comes to the clinic because of odynophagia with solids and liquids and dysphagia that is most severe when eating solid foods. The patient had a past medical history of Shigella colitis last year while she was a Peace Corps volunteer in Peru. She takes oral contraceptives and smokes 1 pack of cigarettes daily. She does not drink alcohol. Vital signs are: temperature 37.8 C (100 F), blood pressure 100/70 mm Hg, pulse 79/min, and respirations 8/min. Physical examination is normal. Electrocardiogram reveals normal sinus rhythms with a rate of 85/min and a markedly enlarged QRS complex in leads V3-V5. Chest x-ray reveals an enlarged cardiac silhouette. A barium esophagram demonstrates a tapering of the distal esophagus that eventually releases as the esophagus is distended. There is no evidence for extrinsic or intrinsic compression of the distal esophagus or an esophageal mass. There is no reflux. The test most likely to lead to a unifying diagnosis in this case is

Top of Form

  A. an agglutination test for trypanosomes
  B. a chest CT
  C. an esophageal manometry
  D. a liver biopsy
  E. a myocardial biopsy

Bottom of Form

Explanation:

The correct answer is A. The findings of achalasia and cardiomyopathy in a patient with history of travel to Central or South America support the diagnosis of Chagas disease. This is caused by infection of Trypanosoma cruzi in the muscles of the heart, esophagus, and colon. A serum agglutination test is a noninvasive means of testing for this infection, and is highly sensitive. Left untreated, heart failure and megacolon could develop.

A chest CT (choice B) would not reveal any specific findings of Chagas disease. Using a CT to evaluate for an occult cancer causing esophageal narrowing is superfluous given the findings on the barium esophagram, which is specific for achalasia.

An esophageal manometry (choice C) is a confirmatory test for achalasia. Given the highly suggestive findings of achalasia on the barium esophagram, manometry would not be necessary.

A liver biopsy (choice D) has no role in the evaluation of Chagas disease.

A myocardial biopsy (choice E) is not necessary to diagnose Chagas disease with the availability of the serum agglutination test for trypanosomes. Invasive procedures should be deferred until they are absolutely necessary.

 

A 29-year-old woman comes to the office because of a 4-day history of pain during urination and increased urinary frequency. She states that her and her husband went away for the weekend, and she developed these symptoms a couple of days after they returned. He is asymptomatic. She had a similar episode on 1 previous occasion, 5 years ago. She does not take any medications and denies any other symptoms. Her temperature is 37.2 C (99.0 F), blood pressure is 110/80 mm Hg, and pulse is 65/min. Physical examination shows mild suprapubic tenderness. There is no costovertebral angle tenderness present. Urinalysis shows:

Color cloudy
Glucose negative
Protein negative
Bacteria present
Nitrites present
WBC 200/hpf
RBC 3/hpf

A urine culture and Gram stain is sent to the laboratory. The most appropriate next step is to

Top of Form

  A. do nothing until the urine culture and Gram stain return
  B. order an intravenous pyelogram
  C. order a renal ultrasound
  D. prescribe a 7-day course of metronidazole
  E. prescribe a 3-day course of trimethoprim-sulfamethoxazole

Bottom of Form

Explanation:

The correct answer is E. This patient most likely has an uncomplicated case of cystitis, which is most likely caused by E. coli spread from the anus to the urethra and up into the bladder. Acute cystitis is a very common infection in women, probably due to the short distance between the anus and urethra. Intercourse and improper wiping techniques (back to front) tend to increase the risk for cystitis in women. Urinalysis and culture are usually obtained. However, treatment can begin based on the symptoms and urinalysis results, even before the cultures are back. A 3-day course of trimethoprim-sulfamethoxazole is the appropriate treatment for uncomplicated cystitis.

It is inappropriate to do nothing until the urine culture and Gram stain return (choice A). You know that she has cystitis based on the history and urinalysis, and therefore an antibiotic is necessary even before the cultures return. Most uncomplicated infections are due to E. coli.

An intravenous pyelogram (choice B) and a renal ultrasound (choice C) are usually only indicated in women with recurrent urinary tract infections (2 in 5 years is not considered recurrent), and in men with urinary tract infections. These tests are done to rule out any obstructions, stones, and renal abnormalities. They are completely unnecessary in a single (the one 5 years ago was so long ago that it does not really count) uncomplicated case of cystitis in a woman.

A 7-day course of metronidazole (choice D) is the treatment for bacterial vaginosis, which is typically characterized by a malodorous vaginal discharge. This patient has cystitis, which is a urinary tract infection, not vaginitis.

 

 

  A 30-year-old previously healthy man is admitted to your medical service for the management of acute renal failure. He had been vacationing in India a week earlier and on arrival back to the United States he developed severe, bloody diarrhea. He has no known sick contacts, is not on any medications, and has no allergies. His temperature is 37.0 C (98.6 F), blood pressure is 100/67 mmHg, pulse is 103/min, and respirations are 23/min. Physical examination is unremarkable. Laboratory studies show:

The most appropriate next diagnostic step is to send a stool culture for

Top of Form

  A. Escherichia Coli O157:H7
  B. Helicobacter pylori
  C. Salmonella typhi
  D. Staphylococcus aureus
  E. Vibrio cholera

Bottom of Form

Explanation:

The correct answer is A. A history of acute renal failure, anemia (hemolytic), and thrombocytopenia following bloody diarrhea is characteristic of the hemolytic-uremic syndrome associated with Escherichia Coli O157:H7 infection.

Helicobacter pylori (choice B) is associated with ulcer disease at the gastric antrum. It is not associated with bloody diarrhea, renal failure, anemia, or thrombocytopenia.

Salmonella typhi (choice C) in its acute form can produce a bloody diarrhea. However, it is not associated with renal failure, anemia, or thrombocytopenia.

Staphylococcus aureus (choice D) is the most common cause of food poisoning in the U.S. However, it is not associated with bloody diarrhea, renal failure, anemia, or thrombocytopenia.

Vibrio cholera (choice E) is associated with massive watery diarrhea. It is not associated with renal failure, anemia, or thrombocytopenia.

 

A 28-year-old man comes to the emergency department complaining of 3 days of non-radiating pain in his right upper quadrant, nausea, and 2 episodes of non-bloody, non-bilious emesis. He also reports that 2 days ago he turned “yellow”. He has no past medical history, has had no recent illnesses, and denies any alcohol or drug abuse. He is married and has not had sexual intercourse with anyone besides his wife in 7 years. His temperature is 37 C (98.6 F), blood pressure is 120/80 mm Hg, and pulse is 65/min. Examination shows scleral icterus and mild jaundice of the skin. There is right upper quadrant tenderness, but no palpable gallbladder or Murphy sign. The laboratory finding most likely to establish the underlying cause of his current symptoms is

Top of Form

  A. positive serum hepatitis A IgG titer
  B. positive serum hepatitis A IgM titer
  C. positive serum hepatitis B surface antibody titer
  D. positive serum hepatitis C antibody
  E. positive serum hepatitis C RNA level

Bottom of Form

Explanation:

The correct answer is B. This patient likely has acute hepatitis A infection. The prodrome of this infection is very similar to this patient’s presentation and within 10-14 days after infection, patients will manifest varying degrees of abdominal pain as well as jaundice. The disease is self- limiting, usually transmitted by contaminated shellfish or oral-anal contact with an infected person or their feces, and does not predispose patients to the same long-term risks as infection with the other hepatitis viruses. In the acute setting, blood titers may be positive for IgM antibody.

A positive serum hepatitis A IgG titer (choice A) would be seen months after the acute infection has passed and is a marker for previous infection.

A positive serum hepatitis B surface antibody titer (choice C) is a marker for previous hepatitis B exposure.

A positive serum hepatitis C antibody (choice D) is a marker for hepatitis C infection. Although both B and C varieties can cause acute viral illnesses similar to this patient, the epidemiology of their transmission is quite different from this patient’s risk factors. Both of these agents are transmitted via blood-to-blood contact and in our day, this is primarily by intravenous drug abuse exposure to infected blood (e.g. healthcare workers and needle sticks).

A positive serum hepatitis C RNA level (choice E) is a test ordered after initial exposure to hepatitis C and in following the activity of disease over time. Unless we suspected hepatitis C infection in this patient, this is not the appropriate test to order.

 

A 44-year-old man is admitted to the hospital from the clinic because of possible sepsis. The patient came to clinic because of left cheek pain that has been increasing for the past 3 days and is associated with rhinorrhea and cough. The patient is HIV positive and takes many HIV medications, but cannot give a more detailed medication history. Vital signs are: temperature 40 C (104 F), blood pressure 100/70 mm Hg, pulse 90/min, and respirations 15/min. Oxygen saturation is 96% on room air. On physical examination, there is tenderness with percussion of the left maxillary and frontal sinuses. The ears, nose, and throat are normal. The lungs are clear. Neurologic examination is significant for a mildly dilated and hyporeactive left pupil. A chest x-ray is normal. A complete blood count, blood cultures, and a basic blood chemistry panel are pending. The next step in the care of this patient is to

Top of Form

  A. administer amoxicillin-clavulanate orally
  B. administer ciprofloxacin, intravenously
  C. administer phenylpropanolamine, orally
  D. administer prednisone, orally
  E. order a CT scan of the head and sinuses

Bottom of Form

Explanation:

The correct answer is E. This immunocompromised patient is at risk for fungal sinusitis. Given the neurological findings, this patient needs imaging to evaluate the extent of disease. This is in preparation for open or endoscopic debridement of the sinuses. Typical causative agents are Aspergillus fumigatus and invasive mucormycosis. Complications of fungal sinus infections include osteomyelitis, meningitis, and brain abscesses.

This patient is at risk for fungal sinusitis, so antibiotics (choice A) would not be the first step in treatment. Amoxicillin-clavulanate is the mainstay in the treatment of and prophylaxis of pneumonia secondary to Pneumococcus carinii in patients with AIDS.

This patient is at risk for fungal sinusitis, so antibiotics such as ciprofloxacin (choice B) would not be the first step in treatment.

An oral decongestant therapy such as phenylpropanolamine (choice C) is often useful in chronic sinusitis, but has no real role in the acute treatment of fungal sinusitis.

Oral steroids (choice D) have no role in the treatment of a fungal infection. Infact, the immunosuppressive properties of the steroid could make the infection worse.

 

A new mother brings her 7-month-old baby boy to the clinic because he has developed “a rash” over his trunk and neck this morning. She states that the baby has had a high fever for the last 4 days, reaching a maximum temperature of 39.5 C (103.1 F). Besides the fever, he has been happy, feeding well, and has not attempted to scratch any of the rash. His temperature is 37.0 C (98.6 F). Physical examination shows a well-developed, well-nourished boy with 2-5 mm, rose-pink macules and papules on the trunk and neck. He has a few small, palpable lymph nodes on his occipital region and moderate erythema on the pharyngeal walls. Conjunctival mucosa is unaffected. The most appropriate next step is to

Top of Form

  A. admit the baby and start work up for Kawasaki
  B. reassure the mother that the baby has mononucleosis and it will resolve on its own
  C. reassure the mother that a roseola rash usually resolves within a few days
  D. tell the mother her baby has chicken pox and only needs supportive care
  E. tell the mother her baby has hand-foot-mouth disease and only needs supportive care

Bottom of Form

Explanation:

The correct answer is C. Roseola, also known as exanthem subitum, is a common, self-limited illness of infancy characterized by 3-5 days of high fever followed by an exanthem after defervescence. Roseola typically develops in children 6 months to 3 years of age. Most infants appear to be well with mild lymphadenopathy. The rash is characteristically flat with rose-pink macules distributed over trunk and neck.

Kawasaki patients (choice A) tend to present with fever longer than 5 days, associated with edema of the hands, conjunctivitis, cervical lymphadenopathy, and morbilliform exanthem with desquamation.

Infectious mononucleosis (choice B) is caused by the Epstein-Barr virus. The disorder begins with headache and malaise with high fevers, which usually lasts 4-14 days. Lymphadenopathy is generalized and the spleen is moderately enlarged in two-thirds of cases. Exanthem of morbilliform type occurs between 4th and 6th day involving the face, trunk, and extremities.

Chicken pox (choice D) is not typically associated with a high fever (mild fever is common) and the exanthem tends to be classic “dew drops on a rose petal” (i.e., vesicles on an erythematous base), at various stages of progression.

Hand-foot-mouth disease (choice E) is characterized by prodrome of low fever, anorexia, sore mouth, malaise, and abdominal pain which precedes the enanthem by 1-2 days. The exanthem occurs shortly after enanthem. The oral lesions tend to begin as small red macules and evolve into small vesicles. One-fourth to two-thirds of affected patients also have highly characteristic vesicular lesions on the hands and feet.

 

  You get a call from the mother of a 5-year-old patient of yours saying that the school nurse called to tell her that her daughter has head lice. The daughter has been scratching her head quite a bit lately, but she thought that it was due to the decreased frequency of shampooing and brushing since she has insisted upon having small braids in her hair. The mother is concerned about her daughter’s health and wants to know what she should do right now because her daughter is still in school. You should advise her that:

Top of Form

  A. Head lice are not responsible for the spread of any disease
  B. Her daughter should be sent home early from school and treated
  C. In addition to infested family members, all members of the household must be treated with a pediculicide
  D. Infestations are influenced by length of hair and frequency of shampooing and brushing
  E. Since there is widespread resistance to permethrin in the United States, lindane is the preferred method of treatment of head lice

Bottom of Form

Explanation:

The correct answer is A. This patient most likely has head lice, which is an infestation caused by Pediculus humanus capitis. It is spread by direct contact with hair of infected persons and sometimes by sharing hats, combs, and hairbrushes. It is treated with permethrin and some believe that a second treatment is advisable 7-10 days after the first. While head lice might be a nuisance, it is not responsible for the spread of any disease

Even though lice is contagious, The American Academy of Pediatrics recommends that children remain in school for the day and that their parents are notified of their condition and properly treat them before sending them to school the next day. Her daughter should be sent home early from school and treated (choice B) is incorrect.

In addition to infested family members, all members of the household must be treated with a pediculicide (choice C) is incorrect. Close contacts and household members should be examined. Treatment is only necessary if the individual is infested. But the bedmates should be treated prophylactically. This is not the same as scabies, where it is recommended that the entire household be treated prophylactically.

Infestations with head lice are NOT influenced by length of hair and frequency of shampooing and brushing (choice D). Transmission occurs by direct contact whether or not the individual has poor hygiene.

While there is some resistance to permethrin in the United States, widespread resistance is found in other countries, and so permethrin is still one of the preferred treatment methods. Lindane is not the preferred method of treatment of head lice (choice E). Lindane is often used for individuals who have not responded to other therapies.

 

A 17-year-old boy who is hospitalized for depression on the general psychiatric unit complains of severe chest pain. The pain is worse on inspiration and has been present for about 2 weeks. His past medical history is significant for depression with multiple suicide gestures for the past 5 years and seasonal allergies. His only medication is fluoxetine. He tells you that he is not sexually active and denies illicit drug use. Review of systems is significant for a recent bronchitis. Vital signs are temperature 37.2 C (99 F), blood pressure 120/70 mm Hg, pulse 92/min, and respirations 10/min. The patient is disheveled, but well developed. Cardiac examination reveals a leathery sound on systole and diastole. There is a normal rate and rhythm, but no third or fourth heart sounds. The lungs are clear. The lower extremities are normal. An electrocardiogram reveals normal sinus rhythm at a rate of 95/min. Chest x-ray reveals moderate cardiomegaly. A prior report from a chest x-ray taken 8 months ago states that the heart size was normal. The next step in managing this patient would be to

Top of Form

  A. begin therapy with ibuprofen
  B. begin therapy with prednisone
  C. obtain permission for an HIV test
  D. order a cardiac perfusion scan
  E. send cardiac enzymes to rule out a myocardial infarction

Bottom of Form

Explanation:

The correct answer is A. This patient is presenting with inspiratory chest pain, a leathery rub on systole and diastole, and a newly enlarged heart. This is consistent with postviral pericarditis, the most common form of pericarditis. Treatment includes nonsteroidal antiinflammatory medication and in refractory cases, steroids. Other causes of pericarditis not apropos to this case include myocardial infarction, connective tissue disease such as rheumatoid arthritis and lupus, drugs such as procainamide, hydralazine, and isoniazid, metastatic cancer such as lung and breast cancer, and renal failure.

Steroid treatment such as prednisone (choice B) is prescribed only after an aggressive course of antiinflammatory treatment has failed given the many side effects of steroid medication.

An HIV test (choice C) is unnecessary given the low-risk social history of this patient. Evaluation and treatment for pericarditis is the most appropriate next step.

A cardiac perfusion scan (choice D) would only be appropriate for a postmyocardial infarction. Evaluation and treatment for pericarditis is the most appropriate next step.

Cardiac enzymes (choice E) are unnecessary given the patient’s age and lack of electrocardiographic findings to suggest a myocardial infarction. This patient is presenting with inspiratory chest pain, a leathery rub on systole and diastole and a newly enlarged heart. This is consistent with postviral pericarditis, the most common cause of pericarditis.

 

A 35-year-old prisoner was recently stabbed in the left leg by another inmate. He is brought to the emergency department by the county corrections officer because of high fevers, swelling of the left thigh, and severe pain at the puncture wound site. His temperature is 38.3 C (101.0 F), blood pressure is 90/50 mmHg, pulse is 120/min, and respirations are 25/min. Although he is awake, he appears lethargic. The left thigh appears pale and swollen around the puncture sight. There is notable crepitus on palpation around the wound. X-rays of the left thigh show translucences in a feathery pattern along the quadriceps. Laboratory studies show:

On exploration of the wound, serosanguinous discharge is noted. Blood cultures are drawn and intravenous crystalloid fluids are instituted. A tetanus booster shot is administered. The most appropriate additional therapy is

Top of Form

  A. ceftriaxone and aztreonam
  B. ciprofloxacin
  C. penicillin and clindamycin
  D. ribavirin and interferon alpha
  E. vancomycin and gentamicin

Bottom of Form

Explanation:

The correct answer is C. This patient most likely has gas gangrene and myonecrosis. The causative organism is the Gram-positive anaerobic cocci, Clostridium perfringens. The x-ray shows classic feathery gas pattern seen in gas gangrene. The triad of management aside from fluid replacement includes antibiotics, surgical debridement, and hyperbaric oxygen. Penicillin and clindamycin are the most active antibiotic regimen against the Clostridia species.

Ceftriaxone (choice A) is an alternative antibiotic choice to penicillin and clindamycin, but aztreonam lacks any significant activity against Gram-positive organisms.

Ciprofloxacin (choice B) and other fluoroquinolones (except levofloxacin) generally have weak activity against the Clostridium species.

Ribavirin and interferon alpha (choice D) are anti-viral agents, not antibacterial agents.

Vancomycin (choice E) has sufficient coverage for C. perfringens. Gentamicin is an aminoglycoside that requires oxygen-dependent transport into bacterial cells, and so it is ineffective against anaerobic organisms.

 

  You are asked to see a baby in the newborn nursery. The baby is small for gestational age and has microcephaly. Physical examination shows hepatomegaly, a widened pulse pressure, a “machinery” heart murmur, and a purpuric skin rash. There is no red reflex in either eye. At this point, you are suspicious that the baby has a congenital infection caused by

Top of Form

  A. Cytomegalovirus
  B. rubella virus
  C. Toxoplasma gondii
  D. Treponema pallidum
  E. varicella-zoster virus

Bottom of Form

Explanation:

The correct answer is B. This baby most likely has congenital rubella. Some of the most common anomalies associated with congenital rubella are intrauterine growth retardation, microcephaly, microphthalmia, cataracts, glaucoma, retinopathy, patent ductus arteriosus, hepatomegaly, jaundice, thrombocytopenia, metaphyseal lucency, and a purpuric rash also known as a “blueberry muffin” rash. Infants may be asymptomatic at birth, but the earlier in pregnancy the mother is infected with the rubella virus, the more likely the baby is to have defects. For example, if a mother is infected in the first 8 weeks of pregnancy, the baby has an 85% chance of having a defect.

Congenital Cytomegalovirus (CMV) infection (choice A) is usually asymptomatic at birth (approximately 85% of the time). Clinical manifestation is found to be severe in approximately 5% of babies with congenital CMV. Manifestations include: intrauterine growth retardation, chorioretinitis, microcephaly, intracerebral calcifications, hepatosplenomegaly, jaundice, thrombocytopenia, neutropenia, purpura, and pneumonia. Although the baby in the question has many of the defects found in congenital CMV, congential heart defects and cataracts are not associated with CMV.

Toxoplasma gondii(choice C) is another organism that can cause congenital infection, but 70–90% of infants with congenital infection are asymptomatic at birth. It is important to note that a large percentage of the infants that are asymptomatic at birth will develop visual impairment, learning disabilities, or mental retardation months to years later. Signs of congenital toxoplasmosis include: hydrocephalus, microcephaly, cerebrospinal fluid abnormalities, intracranial calcifications, chorioretinitis, hepatosplenomegaly, generalized lymphadenopathy, and a maculopapular rash.

Treponema pallidum(choice D) is the organism responsible for syphilis infection. Congenital syphilis is characterized by nonimmune hydrops, prematurity, anemia, neutropenia, thrombocytopenia, pneumonia, and hepatomegaly. Late onset syphilis, which may present up to two years of age, is characterized by snuffles, rash, hepatosplenomegaly, condylomata lata, osteochondritis, cerebrospinal fluid pleocytosis, lymphadenopathy, and thrombocytopenia. Untreated infants may develop late manifestations involving the central nervous system, teeth, eyes, skin, ears, bones, and joints.

Varicella-zoster infection (choice E) in a mother causes different syndromes in a baby depending on the time of the infection. If the mother is infected in the first trimester or early in the second trimester, the baby may develop varicella embryopathy which is characterized by microphthalmia, cataracts, chorioretinitis, cutaneous and bony aplasia/atrophy, and scarring of the skin of the extremity. If the mother is infected during the second 20 weeks of pregnancy, the baby may show no clinical manifestations of varicella, but may develop zoster later in life without ever having extrauterine infection. If the mother develops varicella from 5 days before delivery until 2 days after delivery, the child may develop severe infection, which may lead to death.

 

A 20-year-old man without a significant past medical history comes to the clinic complaining of severe vomiting for the last 10 hours. He denies fevers and reports only 1 episode of small volume, non-bloody diarrhea. He lacks significant abdominal pain. The symptoms reportedly began 6 hours after eating a hamburger and macaroni salad at a neighborhood fast food restaurant. His temperature is 37 C (98.6 F), blood pressure is 105/70 mm Hg, pulse is 100/min, and his respirations are 17/min. He has slightly dry mucous membranes, a non-tender abdomen with decreased bowel sounds, and guaiac-negative stool. The most likely etiology of his gastrointestinal complaints is

Top of Form

  A. Bacillus cereus
  B. Campylobacter jejuni
  C. Clostridium perfringens
  D. E. Coli (0157:H7)
  E. Staphylococcus aureus

Bottom of Form

Explanation:

The correct answer is E. This patient has an illness which is caused by a preformed toxin. S. aureus is a likely culprit in this patient. Staphylococcus produces a preformed toxin, which is obtained from dairy products such as mayonnaise. The illness that follows typically occurs 1-8 hours after ingestion of the toxin. Fever is usually absent and vomiting is major feature of these illnesses. Diarrhea can occur but to a lesser extent than the vomiting. The illness is self limiting and will resolve in 24-48 hours. Treatment involves electrolyte and fluid resuscitation. Antibiotics are not indicated.

Bacillus cereus(choice A) presents in a similar fashion to S. aureus toxin. There is a short incubation period followed by vomiting with a lack of fever. Treatment is identical. The two are differentiated by the types of foods that they are associated with. Bacillus Cereus is typically associated with reheated Chinese fried rice.

Campylobacter jejuni (choice B) causes bloody diarrhea and fevers. Flouroquinolones are indicated in severe cases.

Clostridium perfringes (choice C) causes another toxin based “food poisoning” illness but with different features. This organism is ingested and then produces its toxin while in the gut, thereby causing a longer latent period of 8-16 hours before symptoms begin. Symptoms typically include more abdominal cramping and diarrhea than S. aureus and Bacillus cereus . Treatment is again supportive with resolution in 1-4 days.

Fast food restaurant questions on the USMLE should always make you think about E. Coli 0157:H7 (choice D). This is a strain of E. coli obtained by eating undercooked hamburger meat. There is typically a latent period between 24-72 hours followed by a severe hemorrhagic colitis with fevers and abdominal pain. This patient’s symptoms are not consistent with this organism.

 

  A 27-year-old HIV-positive man comes to the clinic for a periodic health maintenance examination. He contracted the disease 5 years ago from a former partner. He has been followed in the community health clinic since that time. He has no other medical history and takes only diazepam orally for anxiety. His last visit was 11 months ago. His temperature is 37.0 C (98.6 F), blood pressure is 140/85 mm Hg, pulse is 78/min, and respirations are 12/min. He has clear lung fields bilaterally, his skin is free of rashes or excoriations, and his abdomen is soft and nontender. Blood work drawn a few weeks ago reveals a CD4 count of 98 cells/mm3 and a hematocrit of 34% with an MCV of 95 fl. His last tuberculin skin test was 3 months ago and was read as 4mm and flat. In addition to initiating vitamin B12 and folate therapy for his patient, the most appropriate intervention at this time is

Top of Form

  A. antibiotic prophylaxis for PCP pneumonia
  B. antibiotic prophylaxis for tuberculosis
  C. a skin test for tuberculosis
  D. treatment for active tuberculosis infection
  E. none is indicated based upon his CD4 count at this time

Bottom of Form

Explanation:

The correct answer is A. Opportunistic infections occur in people with HIV and define many of the components of the clinical syndrome known as AIDS. They are caused by a wide variety of pathogens and all have the common etiology that the host is susceptible due to the immune destruction brought on by the HIV virus. Prognosis depends on the type of infection and often, even with appropriate therapy, morbidity and mortality is high. There are means to prevent or reduce the likelihood of developing these infections. For this patient, his CD4 count of less than 200 cells/mm3 indicates that he should begin antibiotic prophylaxis, usually with TMP/SMX, for PCP pneumonia.

Starting antibiotic prophylaxis for tuberculosis (choice B) is not routine practice except in persons with a PPD-positive skin test, which this patient does not have.

Since the patient was tested for tuberculosis within the last year, and there is no evidence that he is anergic, there is no indication to test him again at this time (choice C).

The patient does not have active tuberculosis infection (choice D). Even if his PPD test were positive, active infection requires documentation of the organism in sputum by PCR or acid-fast staining.

Any patient with a CD4 count of less than 200 cells/mm3 should be considered for prophylaxis therapy for at least PCP pneumonia and toxoplasmosis (choice E).

 

  A 27-year-old surgery resident comes to the emergency department after lacerating his finger with a scalpel during a routine cholecystectomy. He says that he was suturing the abdominal incision when the scrub nurse told him that it appeared as if there was a cut in his left glove. He immediately ran to the sink, removed his glove, and when he saw the cut he squeezed his finger and held it in bleach for 3 minutes. Since he had only met the patient 20 minutes before the surgery, he does not know of her past medical history. He appears calm, saying that he received the “full Hep B vaccinations series before entering medical school and anyway, the woman does not appear to be an intravenous drug user.” You administer a tetanus vaccine and obtain baseline laboratory studies for HIV, Hepatitis B, and Hepatitis C. An infectious disease specialist happens to be in the emergency department, and you ask her to talk to the resident about the possibility of post-exposure prophylaxis. The surgery resident is pretty confident that the cholecystectomy patient is “clean” and so he goes back to the surgery floor. He returns to the emergency department a few hours later and tells you that the “cholecystectomy woman” admitted to 20 unprotected sexual experiences and a few “experiments” with intravenous drug use years ago. He reluctantly agrees to be treated with zidovudine, lamivudine, indinavir, and interferon. His laboratory studies finally return 1 week later and show:

He comes to your office to discuss the results. At this time the most accurate statement about his condition is:

Top of Form

  A. He has chronic Hepatitis B and can infect sexual partners at this time
  B. He has chronic Hepatitis B but cannot infect sexual partners at this time
  C. His laboratory studies are consistent with immunity from prior Hepatitis B vaccination
  D. His laboratory studies are consistent with immunity from prior Hepatitis B vaccination and infection from the cholecystectomy patient
  E. His laboratory studies are consistent with immunity from prior Hepatitis B vaccination and remote infection with Hepatitis B

Bottom of Form

Explanation:

The correct answer is E. This surgery resident’s laboratory studies are consistent with immunity from prior Hepatitis B vaccination and remote infection with Hepatitis B. An individual that receives the full series of the Hepatitis B vaccine will be Hepatitis B surface antibody positive, which indicates immunity. An individual with a remote Hepatitis B infection will be Hepatitis B core antibody positive, mostly of the IgG class. The Hepatitis core markers are indicative of either infection with or immunity to the nucleocapsid that contains the viral DNA and RNA. This can only be obtained from actual exposure to the virus, not by receiving the vaccine. Patients who receive the Hepatitis B vaccine develop HBsAb without developing HBcAb.

This patient does not have chronic Hepatitis B (choice A and choice B) and therefore cannot transmit the disease. He has antibodies to the virus, both HBcAb and HBsAb, indicating prior infection and prior vaccination. He would have chronic Hepatitis B if he was HBsAg and HBcAb positive, and he would have a high infectivity risk if was HBeAg positive, and low infectivity risk if he was HBeAb positive.

It is incorrect to say that his laboratory studies are consistent with immunity from prior Hepatitis B vaccination (choice C) because patients who receive the Hepatitis B vaccine develop HBsAb without developing HBcAb.

His laboratory studies are not consistent with immunity from prior Hepatitis B vaccination and infection from the cholecystectomy patient (choice D) because of two reasons: the Hepatitis B virus has an incubation period of one to six months (this has only been one week since exposure) and patients who receive the Hepatitis B vaccine develop HBsAb without developing HBcAb. Therefore, this patient’s laboratory studies are consistent with immunity from prior Hepatitis B vaccination and remote infection with Hepatitis B, not with infection from the cholecystectomy patient. An acute infection is characterized by the presence of Hepatitis B surface antigen and Hepatitis B core antibody of the IgM class.

 

A 2-year-old boy is brought to the emergency department by his mother because of a 2-day history of approximately 10 episodes per day of non-bloody watery bowel movements. The child has also had a few episodes of non-bloody, non-bilious emesis. He has not traveled out of the country recently and has not eaten any new foods. His temperature is 38.4 C (101.1 F), blood pressure is 90/50 mm Hg, and pulse is 160/min. The patient weighed 15 kg at his 2-year-old checkup a week ago and presently weighs 13.5 kg. His capillary refill is 2-3 seconds and his mucous membranes are slightly dry. The most appropriate next step in the management of this infant is to

Top of Form

  A. begin intravenous fluid therapy with an isotonic saline solution
  B. begin intravenous fluid therapy with 1/4 NS and 20 mEq KCl/L
  C. begin oral rehydration therapy and obtain a set of electrolytes
  D. get a stool culture and start antibiotics
  E. give the mother instructions regarding oral rehydration and send them home

Bottom of Form

Explanation:

The correct answer is A. Thanks to the 2-year-old check up we have a recent weight on this child and that along with the physical exam and increased heart rate allows us to recognize that this child is about 10% dehydrated. Therefore, he requires some IV fluid therapy. The child should first receive a 20cc/kg bolus of isotonic saline solution and then the rest of the course of the emergency department visit will depend on response to the treatment. Most likely, the child will require another bolus of isotonic solution and then may be placed on a standard IV solution. The parents must then be given strict instructions as to the administration of oral rehydration and if the child improves with the IV fluids and shows that he is capable of being rehydrated orally then they may be discharged home. But the initial treatment is with isotonic saline.

Begin intravenous fluid therapy with 1/4 NS and 20 mEq KCl/L (choice B) is incorrect because those are standard maintenance fluids and should be given only after the boluses of isotonic solution are completed.

The oral rehydration fluid (choice C) being mentioned here is a fluid created by the World Health Organization. It should not be given until after the initial management with isotonic saline and possibly standard maintenance fluids, such as 1/4 NS and 20 mEq KCl/L.

This case appears to be a case of viral gastroenteritis because of the fever and watery stools along with the lack of history suggesting a bacterial cause (travel, eating at a barbecue etc.). Therefore a stool culture (choice D) is not warranted and antibiotics should not be started. Even if the stools were bloody and there was a history suggesting a bacterial cause for the diarrhea (abrupt onset of diarrhea, bloody stool, and no emesis) antibiotics should not be started until a result is obtained from a stool culture. Following are some of the recommendations put out by the Red Book regarding treatment of bacterial gastroenteritis: E. coli – Antibiotics are not necessary unless the diarrhea becomes intractable (>3-4 days) and the isolate is seen to be susceptible to oral antibiotic treatment. Generally the drug of choice is TMP-SMX. Staph aureus – No antibiotic treatment is necessary. Salmonella – Antibiotics are only used for children under 3 months of age, in children with immunodeficiencies, or if there is a positive blood culture. Shigella – Use TMP-SMX if an isolate is recovered and is seen to be susceptible. Campylobacter – If the results of the stool culture come back while the patient is still symptomatic then giving azithromycin may help shorten the duration of the diarrhea and prevent relapse.

Give the mother instructions regarding oral rehydration and send them home (choice E) is incorrect because this child is symptomatic from his dehydration and therefore requires IV therapy and cannot be sent home on oral rehydration therapy alone, though this should be part of the process once the child has received the proper IV fluid therapy.

 

A 22-year-old woman comes to the office because of a 2-day history of vaginal discomfort and mild itching. She has been a long-time patient of yours and has admitted to having 5 lifetime sexual partners. When questioned further, she admits to a frothy vaginal discharge, which is yellowish-green in color. She describes the odor as “fishy”. She says that her symptoms worsen right before onset of menses. Physical examination is unremarkable except for some mild abdominal discomfort. Pelvic examination shows a “frothy” vaginal discharge and a friable cervix with numerous petechiae. The most likely diagnosis is

Top of Form

  A. Bacterial vaginosis
  B. Candida albicans
  C. normal vaginal discharge
  D. Trichomonas vaginalis

Bottom of Form

Explanation:

The correct answer is D. The clinical picture in this patient is consistent with Trichomonas vaginalis. Although the infection is often asymptomatic, men may develop urethritis, and women may complain of a frothy vaginal discharge that is greenish-yellow in color that may have a fishy odor. Women may also have some lower abdominal tenderness with more severe symptoms just before or just after menstruation. On exam, the vaginal mucosa may be erythematous with an inflamed and friable cervix. Women may also have a “strawberry cervix”, a term used when there are multiple petechiae on the cervix. Although a wet prep is positive in only 40-80% of the cases, and therefore not necessary for diagnosis, seeing the trichomonads with their flagella and jerky motility is diagnostic.

Bacterial vaginosis (BV) (choice A) is a syndrome seen in sexually active females which is usually asymptomatic but often presents with a thin, white, foul smelling discharge which people often describe as “fishy”. BV is usually not associated with pruritus, dysuria, or abdominal pain. BV is the most prevalent vaginal infection in sexually active females. Although not completely clear, causes are thought to include Gardnerella vaginalis, Mycoplasma hominis, and anaerobic bacteria. In order to diagnose BV, a woman must have at least 3 of the following: a whitish gray non-inflammatory vaginal discharge that adheres to the vaginal wall, a vaginal fluid pH greater than 4.5, a “fishy” odor to the vaginal fluid either before or after mixing with 10% potassium hydroxide, and a wet mount that shows “clue” cells, which are epithelial cells with smudged borders due to bacteria adherent to the cell membrane

Yeast infections are caused by overgrowth of Candida albicans(choice B) often due to factors such as pregnancy, antibiotic use, diabetes, and oral contraceptive use. Some women report predisposition to yeast infections immediately preceding menstruation. These women usually present with complaints of intense pruritus and burning accompanied by a thick white “cottage cheese”-like vaginal discharge. In a woman with a yeast infection, you would expect to see yeast and pseudohyphae on a wet mount.

Normal vaginal discharge (choice C) is usually a scant to moderate amount of clear-white colored discharge. There is usually no strong odor present. There are many lactobacilli and normal epithelial cells.

 

A 6-year-old girl is brought to the clinic because of a 24-hour history of an ” itchy, red rash.” Over the past 7 days she has not been feeling well. She had a fever reaching 39.3 C (102.8 F), a headache, and muscle aches. Her mother treated her with acetaminophen and these symptoms resolved. Now she has this rash that appeared over night, as the other symptoms resolved. Her temperature is 37 C (98.6 F). Physical examination shows an erythematous facial rash on the cheeks and a symmetric, maculopapular, lace-like rash on the arms, buttocks, and thighs. The remainder of the examination is unremarkable. Laboratory studies show:

At this time the most correct statement about her condition is:

Top of Form

  A. Droplet precautions should be used to prevent the spread of infection
  B. Intravenous immunoglobulin therapy should be given
  C. Pregnant women should not have contact with this patient
  D. She should return to school after this office visit
  E. She should be given a blood transfusion to prevent an aplastic crisis

Bottom of Form

Explanation:

The correct answer is D. This patient has the classic symptoms of an erythema infectiosum (fifth disease), which is an infection caused by Parvovirus B19. Patients with this disease are only infectious before the onset of the rash, during the period with the nonspecific febrile illness. The virus typically only causes a significant, severe illness in individuals with sickle cell disease and other hemoglobinopathies. In rare cases, has parvovirus, during pregnancy, been associated with fetal hydrops and death. But as stated earlier, this patient can go back to school because she is no longer contagious.

Droplet precautions used to prevent the spread of infection (choice A), is unnecessary at this stage in the disease because this patient is no longer infectious. Droplet precautions are used in hospitals for those caring for patients with diseases that are transmitted through droplets containing microorganisms. They are used to prevent the host from catching the infection if the infected patient coughs or sneezes on them or during procedures such as suctioning or bronchoscopy. Droplet precautions require a mask if within 3 feet of the patient and or in a private room. In addition to standard precautions, parvovirus B19 requires droplet precautions for those caring for these hospitalized patients.

Intravenous immunoglobulin therapy (choice B) is given to immunocompromised patients with chronic parvovirus infection. Supportive care is all that is indicated for this patient.

Since this patient is no longer infectious, it is incorrect to say that pregnant women should not have contact with this patient (choice C).

Parvovirus B19 is associated with thrombocytopenia, neutropenia, and red blood cell aplasia usually in patients with sickle cell disease and other hemoglobinopathies. Transfusions are used in these patients with aplastic crises. Transfusions are not routinely given in previously healthy children who have parvovirus B19 infection to prevent aplastic crises. Since this patient’s complete blood count is normal, she should NOT be given a blood transfusion to prevent an aplastic crisis (choice E).

 

A 33-year-old HIV-positive man with a CD 4 count of 125 comes to the clinic with multiple bumps on his face, requesting that they be removed by liquid nitrogen. He states he has had multiple molluscum lesions which recur every so often and his previous doctors have removed them with the “freezing” technique. He recently discontinued all of his antiretroviral medications due to lack of motivation and was referred for psychiatric evaluation. He reports that in the past 2 months he has experienced frequent low-grade fevers, headaches, and myalgias. His temperature is 38.1 C (100.6 F), blood pressure is 110/70 mm Hg, pulse is 70/min, and respirations are 16/min. He appears cachetic with temporal wasting. There are over 20 papules with central umbilication measuring 0.2-1.5 cm in diameter on his face. In most of the lesions, the central dimples are covered with a hemorrhagic crust. Superficial ulcerations are appreciated on oral mucosa. Multiple 0.5-1.0 cm mobile lymphadenopathy are present in cervical and supraclavicular regions. The most appropriate next step in management is to

Top of Form

  A. biopsy one of the lesions to rule out cutaneus cryptococcosis
  B. give him acyclovir and ask him to come back in 2 months
  C. tell him there is no medical indications to treat these lesions, which are of cosmetic concern
  D. diagnose this as tinea faciei and start treatment with topical terbinafine
  E. treat the lesions with liquid nitrogen and have patient return to clinic as needed

Bottom of Form

Explanation:

The correct answer is A. Cutaneous cryptococcosis is primarily an opportunistic infection with Cryptococcus neoformans which affects the central nervous system. The disease is subacute and mild headaches, fever, and malaise are the predominant features. Cutaneous cryptococcosis in HIV infection usually presents as small papules with central umbilication covered with hemorrhagic crust. Tumors and ulcerative lesions of skin or mucous membranes are also seen. Often, the lesions resemble Molluscum contagiosum, herpes infection, and Kaposi sarcoma. Whenever presented with an immunosuppressed patient with lesions resembling molluscum, always bear in mind other deep fungal infections. If there is enough doubt, biopsy the lesions for culture and warn the laboratory that cryptococcus is suspected, since the culture form is highly contagious.

Acyclovir (choice B) is incorrect because herpes may be in the differential of cutaneous cryptococcus. One should rule out more dangerous infection first before treating blindly.

No treatment (choice C) is incorrect because, as mentioned above, immunosuppressed patients are at a much higher risk for deep fungal infection and this patient’s CD4 count is well under 200.

Tinea faciei (choice D) is incorrect because this is a superficial fungal infection that usually does not result in papules and is not associated with systemic symptoms such as fever and malaise as in this patient.

Treating the lesions with liquid nitrogen (choice E) as if these are typical Molluscum contagiosum lesions is incorrect as described above.

 

A 3-week-old infant is brought into the office by her mother for a newborn examination. Her birth was without incident; however she received ampicillin and gentamicin for 2 days after birth for rule-out sepsis. The blood and cerebrospinal fluid cultures were negative and she was therefore discharged home with the mother on day of life 3. She has been breast fed exclusively and is feeding every 3 hours without difficulty. She is voiding and stooling regularly. The mother notes, however, that for the past 1 weeks she has had reddened skin over the diaper area and cries frequently until her diaper is changed. She has tried using zinc oxide cream at every diaper change, but the rash has persisted. On physical examination, you note a bright red eruption over the perineal area, which involves the intertriginous areas. There are sharp borders with pinpoint satellite papules and scattered pustules. The remainder of the exam is unremarkable. KOH preparation of the pustular material reveals pseudohyphae and spores. The most appropriate therapy at this time is

Top of Form

  A. continue zinc oxide
  B. frequent exposure to air
  C. intravenous cephalexin
  D. oral fluconazole
  E. topical acyclovir
  F. topical nystatin

Bottom of Form

Explanation:

The correct answer is F. This infant has candidal diaper dermatitis, which is a common occurrence in infants after receiving antibiotic therapy. It has a characteristic bright red appearance with satellite lesions and a sharply demarcated border. KOH preparation reveals the classical pseudohyphae and budding yeasts that are characteristic of candida. Topical antifungals, such as nystatin, are the treatment of choice.

Zinc oxide (choice A) and frequent exposure to air (choice B) are appropriate therapies for irritant diaper dermatitis, which is caused by harsh soaps, detergents, and the ammonia-laden environment within the diaper. This dermatitis is also erythematous, but classically spares the intertriginous creases.

Intravenous cephalexin (choice C) is inappropriate because there is no indication of bacterial infection requiring IV antibiotics.

Oral fluconazole (choice D) is not indicated since this infection responds very well to topical antifungals.

Topical acyclovir (choice E) is incorrect as the etiology of this infection is not viral.

 

A previously healthy 20-year-old man comes to his college medical clinic for headaches and low-grade fevers. He is discharged home with the diagnosis of a “viral syndrome” and instructed to get ample rest. Approximately three hours later his roommate calls 911 reporting that his friend is unconscious and not arousable. On arrival the paramedics find a lethargic, febrile man lying on the floor and unresponsive. The patient is stabilized and he is rushed emergently to the local hospital where an abdominal CT scan shows bilateral adrenal hemorrhages. His blood pressure is 80/40 mm Hg and his pulse is 110/min. He appears very ill and continues to be non-responsive. The most appropriate study at this time is a/an

Top of Form

  A. Cort-Stim test
  B. cortisol level
  C. high-dose ACTH stimulation test
  D. insulin response test
  E. low-dose ACTH stimulation test

Bottom of Form

Explanation:

The correct answer is B. This patient has bilateral adrenal hemorrhages in the setting of fulminant disseminated Neisseria sepsis. This is called the Waterhouse-Friderichsen Syndrome. The patient therefore almost certainly has acute adrenal insufficiency. In a critically ill patient, a random cortisol level that is low confirms this diagnosis.

A Cort-Stim test (choice A) is a test whereby synthetic ACTH is administered and the cortisol response is measured one hour later. It is more sensitive at detecting adrenal insufficiency but is both too time-consuming and unnecessary in this patient. The threshold for supplying exogenous steroids is very low so all that is needed is a suspicion for insufficiency that a random cortisol level can easily provide.

A high-dose ACTH stimulation test (choice C) or low-dose ACTH stimulation test (choice E) are both very sensitive tests that require 24 hours to perform. They both are useful at quantifying adrenal function so that differentiation between various causes of adrenal insufficiency can be made. However, they are not the most appropriate studies in this acute situation.

The insulin response test (choice D) is another method used to indirectly assess adrenal function but will not be useful in this patient since the goal is rapid direct assessment of her adrenal function.

 

A 71-year-old man with osteoarthritis comes to the office complaining of a painful “band-like” rash across his left chest. He denies ever having a similar rash before. He plays golf 3 times per week and takes only nonsteroidal antiinflammatory agents for pain from his arthritis. His temperature is 37.0 C (98.6 F). On his left chest, in the T5 dermatomal distribution, is a macular-papular, erythematous rash that is painful to the touch. There is mild weeping of some of the papules. The most appropriate therapy is at this time is

Top of Form

  A. antibiotics
  B. antifungal agents
  C. corticosteroids
  D. ganciclovir
  E. gabapentin

Bottom of Form

Explanation:

The correct answer is D. The patient has herpes zoster, also known as shingles. The disease is a result of reactivation of latent varicella zoster virus in the dorsal root ganglia. The disease follows a dermatomal distribution and is very painful. The goals of therapy are to hasten the resolution of the symptoms and to prevent the development of postherpetic neuralgia, an often crippling neuropathic pain disorder resulting from the shingles infection. The drugs best able to accomplish both of these goals are the oral antiretroviral drugs, ganciclovir, acyclovir, and famciclovir.

Neither oral antibiotics (choice A) nor oral antifungal agents (choice B) have any role in the treatment of this disease, since it is a reactivation of a viral infection. Unless there is evidence of a superinfection with a bacterial or fungal etiology, these classes of drugs should be avoided.

The use of corticosteroids (choice C) for this disease will exacerbate the symptoms. Although dermatologists liberally utilize steroid therapy, its use in this case acts as an immunosuppressive agent and will exacerbate the primary manifestations of rash and pain.

Gabapentin (choice E) is used for the treatment of postherpetic neuralgia and other neuropathic pain syndromes. Gabapentin is structurally related to the neurotransmitter GABA (gamma-aminobutyric acid) but it does not interact with GABA receptors, as it is not converted metabolically into GABA or a GABA agonist, and it is not an inhibitor of GABA uptake or degradation. It has no utility in treatment of acute shingles.